Sei sulla pagina 1di 99

Romanian Mathematical Society-Mehedinți Branch 2019

ROMANIAN MATHEMATICAL
SOCIETY
Mehedinți Branch

ROMANIAN MATHEMATICAL MAGAZINE

R.M.M.

Nr.22-2018

1 ROMANIAN MATHEMATICAL MAGAZINE NR. 22


Romanian Mathematical Society-Mehedinți Branch 2019

ROMANIAN MATHEMATICAL
SOCIETY
Mehedinți Branch
DANIEL SITARU-ROMANIA EDITOR IN CHIEF
ROMANIAN MATHEMATICAL MAGAZINE-PAPER VARIANT
ISSN 1584-4897
GHEORGHE CĂINICEANU-ROMANIA
DAN NĂNUȚI-ROMANIA
EMILIA RĂDUCAN-ROMANIA
MARIA UNGUREANU-ROMANIA
DANA PAPONIU-ROMANIA
GIMOIU IULIANA-ROMANIA EDITORIAL BOARD
DRAGA TĂTUCU MARIANA-ROMANIA
CLAUDIA NĂNUȚI-ROMANIA
DAN NEDEIANU-ROMANIA
GABRIELA BONDOC-ROMANIA
OVIDIU TICUȘI-ROMANIA
ROMANIAN MATHEMATICAL MAGAZINE-INTERACTIVE JOURNAL
ISSN 2501-0099 WWW.SSMRMH.RO
DANIEL WISNIEWSKI-USA
VALMIR KRASNICI-KOSOVO EDITORIAL BOARD
ALEXANDER BOGOMOLNY-USA

2 ROMANIAN MATHEMATICAL MAGAZINE NR. 22


Romanian Mathematical Society-Mehedinți Branch 2019

CONTENT

Algebraic inequalities from geometrical configurations- Daniel Sitaru,Claudia


Nănuți........................................................................................................................................................................4

The beauty of inequalities-Do Huu Duc Thinh........................................................................................7


Elemente de geometria triunghiului in coordonate baricentrice-Neculai Stanciu...............20

Few applications of Cocea’s inequality- Daniel Sitaru,Claudia Nănuți...........................................25

About problem UP.147-ROMANIAN MATHEMATICAL MAGAZINE-Marin Chirciu..……………28

About problem 3677-CRUX MATHEMATICORUM- Marin Chirciu……. ……………………………….32

About problem JP.173-ROMANIAN MATHEMATICAL MAGAZINE- Marin Chirciu................35

About INEQUALITY IN TRIANGLE-884-ROMANIAN MATHEMATICAL MAGAZINE- Marin


Chirciu.......................................................................................................................................................................37

Plynomes and matrices-Marian Ursărescu………………………………………………………………………39

Proposed problems………………………………………….………………………………………………………………...41

Index of proposers and solvers RMM-22 Paper Magazine.………………………………………….…………99

3 ROMANIAN MATHEMATICAL MAGAZINE NR. 22


Romanian Mathematical Society-Mehedinți Branch 2019

ALGEBRAIC INEQUALITIES FROM GEOMETRICAL CONFIGURATIONS


By Daniel Sitaru, Claudia Nănuți – Romania
Abstract: In this article we build geometrical configurations and extract algebraic inequalities
from geometrical properties.

Aplication 1 : If , , ≥ 0 then:

1.1. + + √2 + + + √2 ≥ +

1.2 + + √ + + ≥ + + √2

1.3 + + √2 + + ≥ + + √2

Solution: Let be Δ such that: = ; = ; = ; = ;

3
= =
4

By cosine theorem: = + ; = + + √2; = + + √2

In Δ : + > ; + > ; + >

+ + √2 + + + √2 ≥ +

+ + √2 + + ≥ + + √2

+ + √2 + + ≥ + + √2

Equality holds for = = = 0.

4 ROMANIAN MATHEMATICAL MAGAZINE NR. 22


Romanian Mathematical Society-Mehedinți Branch 2019
Aplication 2: If , , ≥ 0 then:

2.1. + +√ + + ≥ + + √3

2.2. + + + + √3 ≥ √ + +

2.3. √ + + + + + √3 ≥ +

Solution: Let be Δ such that: = ; = ; = ; = ; = ;

5
=
6

By cosine theorem: = + ; =√ + + ; = + + √3

In Δ : + > ; + > ; + >

+ + + + ≥ + + √3

+ + + + √3 ≥ + +

+ + + + + √3 ≥ +

Equality holds for = = = 0.

Aplication 3 : If , , ≥ 0 then:

3.1. + + √3 + + + √3 ≥ + +

5 ROMANIAN MATHEMATICAL MAGAZINE NR. 22


Romanian Mathematical Society-Mehedinți Branch 2019

3.2 + + √3 + + + ≥ + + √3

3.3. + + √3 + + + ≥ + + √3

Solution: Let Δ such that: = ; = ; =

5
= 60°; = =
6

By cosine theorem: = + + ; = + + √3; = + + √3

In Δ : + > ; + > ; + >

+ + √3 + + + √3 ≥ + +

+ + √3 + + + ≥ + + √3

+ + √3 + + + ≥ + + √3

Aplication 4 : If , , ≥ 0 then:

+ + + + + ≥ 2 3( + + )

Solution:

6 ROMANIAN MATHEMATICAL MAGAZINE NR. 22


Romanian Mathematical Society-Mehedinți Branch 2019
Let be a tetrahedron such that: ⊥ ⊥ ⊥ ; = ; = ; =

+ +
= ; = + ; =
+ + +

⋅ 1 + +
[ ]= = ⋅ + ⋅
2 2 +

+ +
[ ]=
2

By Mitrinovic’s inequality in Δ : ≥ √3 ⇒ ≥ √3 ⇒ ≥ √3

+ + + +√ + √3
≥ ⋅ + +
4 2

+ + + + + ≥ 2 3( + + )

[1] Mihály Bencze; Daniel Sitaru – “699 Olympic Mathematical Challenges” – “Studis”
Publishing House – Iași – 2017
[2] Mihály Bencze; Daniel Sitaru – “Quantum Mathematical Power” – “Studis” Publishing
House – Iași – 2017
[3] Mihály Bencze; Daniel Sitaru – “Olympic Mathematical Energy” – “Studis” Publishing
House – Iași – 2018
[4] George Apostolopoulos; Daniel Sitaru – “The Olympic Mathematical Marathon” –
“Cartea Românească” Publishing House – Pitești – 2017
[5] Romanian Mathematical Magazine – Interactive Journal – www.ssmrmh.ro
THE BEAUTY OF INEQUALITIES

By Do Huu Duc Thinh-Vietnam


Inequality is one of the nicest and hardest major parts of Mathematics, because of its
appearance in Math contests; and also it requires a solid knowledge to solve an inequality
problem. But because of the elegance, Inequality attracts lots of student generations joining
in solving and creating more. To maintain the passion and creativeness, many sites and
forums nowadays have shared lots of topics and documents of inequality for everybody, but
it’s still intricate and note arranged entiringly. Maybe many people will consider that
studying Inequality is boring, since its large amount of knowledges and complex
presentation of that amount of knowledges, especially upper secondary high school and
7 ROMANIAN MATHEMATICAL MAGAZINE NR. 22
Romanian Mathematical Society-Mehedinți Branch 2019
higher. But I will demonstrate carefully and clearly for everyone through this document file,
so everyone can realize how beautiful inequality is and its application. This file that I make is
based on documents of many teachers in Vietnam and around the world, such as: Tran Quoc
Anh, Nguyen Van Mau, Pham Kim Hung, Vo Quoc Ba Can, Vasile Cîrtoaje,… I spent lots of
time to compile the file so if there’s a mistake, I hope everybody can understand and give
me feedback because my knowledge is limited. All feedbacks can be sent to the following
email: thinh06032001@gmail.com or Facebook. Thank you very much!
Season 1: Some old and modern techniques of Inequalities in Math contests
In this season, I will state again some inequalities and basic techniques that are useful to find
the solution of proving the inequality and solving Min-Max problems. Also I will add more
lemmas with inferences and developments so everyone, especially students, can find a nice
way to solve the inequality.
I. Inequalities can be proved by equivalence:
( )
1) + ≥ ≥2 (Basic BCS inequality for 2 numbers)

→ ≥ ≥√ ≥ = (Basic RMS-AM-GM-HM inequality for , > 0)

→ ≤ ≤ + − with , ≥ 0 such that + >0


( )
2) + + ≥ ≥ + +
∗ 3( + + )≥( + + ) ≥3 ( + + )

∗ + + ≥ 3( + + )≥ + + ≥ 3( + + ) , , >0

3) + + ≥ 2 + 2 − 2
4) + + ≥ 3 ∀ , , such that + + ≥ 0 (special case: , , > 0 –
Cauchy’s inequality for 3 non-negative numbers)
5) ( + )( + ) ≥ ( + ) (Basic Bunyakovsky’s inequality)
6) + ≥ ( + ) ∀ , such that + ≥ 0
→ 4( + ) ≥ ( + ) ≥ 4 ( + ) for + ≥ 0
7) 2( + − ) ≥ + ≥ 2 (2 − 3 + 2 )∀ ,
→√ − + ≥ = + − with + ≠0
8) + ≥( )
with real numbers , such that > 0 (special case: , > 0)
9) + + ≥ with , , > 0 (Nesbitt’s inequality) → This is even true for real
numbers ; ; such that + + > 0.
10) ( + )( + )( + ) ≥ ( + + )( + + ) ≥ 8 for , , ≥ 0.
∗ ≥ ( + − )( + − )( + − ) , ,

∗( + + ) ≥ 27 , , ≥ 0 ( − )
( )
11) + ≥ with , > 0 (Basic Bunyakovsky Cauchy-Schwarz inequality)
* Also, from this inequality we have the chain: + ≥ 2( + )≥ + ≥ 2√ for
, > 0.
∗ 2( + ) ≥ ( + )( + ) ≥ 2( + ) ℎ ≥ , ≥
12) (
(Basic
∗3 + + ) ≥ ( + + )( + + ) ≥ 3( + + ) ℎ ≥ ≥ , ≥ ≥
Chebyshev’s inequality)

8 ROMANIAN MATHEMATICAL MAGAZINE NR. 22


Romanian Mathematical Society-Mehedinți Branch 2019
13) + + ≥ + + with ≥ ≥
* More general: + + ≥ + + for ≥ ≥ ≥ 0, ∈
14) 2( + ) ≥ ( + )( + ) with , > 0; , , ∈ : + =
→ + ≥ + for , ≥ 0, , ∈
15) + + ≥ ( + ) ; − + ≥ ( + ) ∀ , → 3 ≥ ≥ with
real numbers , : + > 0.
16) √ + + √ + ≥ ( + ) + ( + ) (Basic Minkovsky’s inequality)
17*) +( ) ≥ with , ≥ 0 (The equality happens iff = = 1)
( )^
18) Consider ( ; ) = + − with , > 0. If ≥ 1 then ( ; ) ≥ 0; if
≤ 1 then ( ; ) ≤ 0.
19) + ≥ with , > 1
20) ( + + )( + + ) ≥ 3. max{ + + ; + + } with
, , ≥0
21) ( + + ) ≥ ( + + )( + + ) ∀ , , > 0
22) + + ≥ ≥ + + with ≥ ≥ > 0
23*)( + + ) ≥ 3( + + ) (Vasile’s inequality)
24) ≥ 1 − with ≥ 0 → + + ≥ ∀ , , ≥ 0: + + = 3.
25) ( + + ) ≥ + + + 24 ∀ , , > 0
II. Some familiar Inequalities, lemmas and techniques: (ascending by higher level)
a) For junior – early – senior:
⋯ ⋯
1) ≥ ≥ … ≥ with > 0 ∀ = 1; 2; … ;

and ∈ , ≥ 2
(RMS-AM-GM-HM inequality for positive numbers)
→ If ; ; … ; are positive real numbers that + … + = 1 then with same
condition for we have:
+ +⋯+ ≥ ⋅ … (Weighted AM-GM inequality)
2) For real numbers ; ( = 1; 2; … ; ) we have:
( + + ⋯ + )( + + ⋯ + ) ≥ ( + + ⋯+ )
( ⋯ )
(Cauchy-Schwarz inequality) → + + ⋯+ ≥ ∀ >0

→ + + ⋯+ + ≥ ( + + ⋯+ )≥ + + ⋯+ ;∀ >0
3) Let ≥ ≥⋯≥
∗ ≥ ≥⋯≥ ℎ : ( + + ⋯+ )≥( + +⋯+ )( + + ⋯+ )
∗ ≤ ≤⋯≤ ℎ : ( + +⋯+ )≤( + + ⋯+ )( + + ⋯+ )
(Chebyshev’s inequality)
4) If . ≥ 0 ( = 1; 2; . . ; , = 1; 2; … ; ) then we have:
( . + . + ⋯ + . )( . + . + ⋯ + . ) … ( . + . + ⋯ + . )≥
≥ . . … . + . . … . + ⋯+ . . … .
(Hölder’s inequality)
Ex: ∗ ( + )( + ) ≥ √ +√ ∀ , , , ≥0

9 ROMANIAN MATHEMATICAL MAGAZINE NR. 22


Romanian Mathematical Society-Mehedinți Branch 2019

∗( + + )( + + )( + + )≥ √ + + ∀ , , , , , , , , ≥ 0
∗ + + ⋯+ ≥ + + ⋯+ > 0 ( = 1; 2; … ; ); ∈ , ≥2

+ +⋯+ + +⋯+
→ ∗ ≥ ℎ ( )

1 1 1 1 1 1
∗ + + ⋯+ ≥ > 0 ( = 1; 2; … ; ) → + + ⋯+ ≥
( + +⋯ ) ( + + ⋯+ )
→ Let and > 0( = 1; 2; … ; ) and real , > 0 such that + = . Then we have:
+ +⋯+ ⋅ + +. . + ≥ + +⋯+ (General Hölder’s
inequality)
5) Let , > 0 ( = 1; 2; … ; ) and any > 1. Then we have:
+ + …( + ) ≥ ( + + ⋯+ ) + ( + +⋯+ )
(Minkovsky’s inequality) → similarly for 3 variables , , > 0.
∗ (1 + ) ≥ 1 + ≥ 1 ≤0
6) For any ≥ −1 we have: (Bernoulli’s
∗ (1 + ) ≤ 1 + 0 ≤ ≤ 1
inequality)
7) For any positive integer and , , ≥ 0 we have:
( − )( − ) + ( − )( − ) + ( − )( − ) ≥ 0
(Schur’s inequality) → This is also true for real ≥ 1 and equality happens iff = = or
( ; ; )~(0; ; ) with > 0.
Δ Case = 1 – Schur deg 3: All forms: ( , , will be discussed in later part)
∗ + + +3 ≥ ( + )+ ( + )+ ( + ) →( + + ) +9 ≥
9
≥ 4( + + )( + + ) → + + + ≥ 2( + + ) →
+ +
→ ( + + )( + + + + + ) ≥ 3[ ( + ) + ( + ) + ( + )]
∗ ≥ ( + − )( + − )( + − ) (Well – known result)
∗( − ) ( + − )+( − ) ( + − )+( − ) ( + − )≥0
∗ 3( + + ) ≥ ( + + )[2( + + ) − − − ]
∗4 ( + + ) + 15 ≥ ( + + )
* + + + ( )( )( ) ≥ 2
∗ + + +3 ≥
+ + + + + for , , > 0: = 1.
∗( + + )( + + )≥( + + )( + + )
Δ Case = 2 – Schur deg 4: All forms:
∗ + + + ( + + )≥ ( + )+ ( + )+ ( + )→
→ + + +2 ( + + ) ≥ ( + + )( + + )
6 ( + + ) 9
∗ 2( + + ) − ( + + ) ≤ ≤
+ + + + + + +
* [( − )( + − )] + [( − )( + − )] + [( − )( + − )] ≥ 0
→ Let , , , , , ≥ 0. Then we have:
( − )( − ) + ( − )( − ) + ( − )( − ) ≥ 0 iff ≥ ≥ and:
∗ ≥ ⋎ ≥ ∗ + ≥
∗ + ≥ ∗ √ + √ ≥
∗ ≥ ⋎ ≥
(General Vornicu – Schur inequality)
10 ROMANIAN MATHEMATICAL MAGAZINE NR. 22
Romanian Mathematical Society-Mehedinți Branch 2019

8) ( )
+( )
+( )
≥ ( )
for , , ≥ 0, no 2 of which are 0. (Iran 96
inequality)
9) ( + + ) ≥ 3( + + ) (Vasile’s inequality) → The equality happens iff
= = and also for ( ; ; ) = ⋅ sin ; ⋅ sin ; ⋅ sin or any cyclic
permutation.
10) Let and ( = 1; 2; … ; ) such that:
∗ ≥ ≥⋯≥ ≥ 0, ≥ ≥⋯≥ ≥0
∗ ≥ ; + ≥ + ;…; + + ⋯+ ≥ + +⋯+
∗ + +⋯+ + = + + ⋯+ +
For ≥ 0 we have: ∑ … ≥∑ … , where ( ; ; … ; ) are all
the permutations of (1; 2; … ; )
(Muirhead’s inequality)
E.g: + ≥ ( + )→ + ≥ +
+ + ≥ + + → + + ≥ + +
+ + ≥ ( + + )→ + + ≥
≥ + +
b) For senior and higher classes: (it’s very hard to express the real form of these inequalities
so I will try my best.)
≥ ≥⋯≥ ; ≥ ≥⋯≥
1) ∗ If and ( ; ; … ; ) is an arbitrary
≤ ≤⋯≤ ; ≤ ≤⋯≤
permutation of (1; 2; … ; ) then: + + ⋯+ ≥ + + ⋯+ .
∗ If ≥ ≥⋯≥ and ≤ ≤⋯≤ then:
+ + ⋯+ ≤ + +⋯+ ;
( + + ⋯+ ) ≤ ( + + ⋯ + )( + + ⋯ + )
(Rearrangement inequality)
2) * Convex function: If , ≥ 0 such that + = 1 then ( ) is called a convex function on
( ; ) ⊂ iff ∀ ; ∈ we have: ( + )≤ ( )+ ( )
* Concave function: If , ≥ 0 such that + = 1 then ( ) is called a concave function on
( ; ) ⊂ iff ∀ ; ∈ we have: ( + )≥ ( )+ ( )
* If ( ) is a convex function on interval ⊂ then for any ∈ ( = 1; 2; … ; ) we have:
⋯ ( ) ( ) ⋯ ( )
≤ (Classic Jensen’s inequality)
* If ( ) is a convex function on interval ⊂ then for any ∈ ( = 1; 2; … ; ) and
> 0 we have:
( )+ ( )+ ⋯+ ( ) + + ⋯+

+ +⋯+ + +⋯+
And if ( ) is a concave function then the inequality is reversed. (General Jensen’s
inequality)
→ ∗ If ( ) is a convex and continuous function on interval ⊂ then for any
∈ ( = 1; 2; … ; ) and ∈ (0,1) such that + + ⋯ + = 1 we have:
( )+ ( )+ ⋯+ ( )≥ ( + +⋯+ ). And if ( ) is a concave
function then the inequality is reversed.
* The classic inequality is a special case from the general one with = =⋯=
( )
* Let and = 1; 2; . . ; ∈ ⊂ ( ) such that:
_ ≥ ≥⋯≥ ; ≥ ≥⋯≥

11 ROMANIAN MATHEMATICAL MAGAZINE NR. 22


Romanian Mathematical Society-Mehedinți Branch 2019
_ ≥ ; + ≥ + ;…; + + ⋯+ ≥ + +⋯+
_ + +⋯+ + = + + ⋯+ +
If ( ) is a convex function on then we have:
( ) + ( ) + ⋯ + ( ) ≥ ( ) + ( ) + ⋯ + ( ) (Karamata’s inequality)
* If ( ) is a convex function on ⊂ then for ∈ ( = 1; 2; … ; ) we have:
+ +⋯+
( ) + ( ) + ⋯ + ( ) + ( − 2) ≥

≥ ( − 1)[ ( ) + ( ) + ⋯ + ( )] where + = ( = 1; 2; … ; )
(Popoviciu’s inequality)
3) Define = + + , = + + , = with , , are any real numbers. If
= − 3 then we have: ≤ ≤
→ The minimum and maximum happens iff 2 of 3 variables ; ; are equal.
4) * Let ( ; ; ) be a symmetric polynomial of degree 3 with , , ≥ 0. Then:
( ; ; ) ≥ 0 ⇔ (1; 1; 1); (1; 1; 0); (1; 0; 0) ≥ 0 (SD3 theorem)
* Let ( ; ; ) be a cyclic homogeneous polynomial of degree 3 with , , ≥ 0. Then:
( ; ; ) ≥ 0 ⇔ (1; 1; 1) ≥ 0; ( ; ; 0) ≥ 0 (CD3 theorem)
→ Let ( ; ; ) be a cyclic homogeneous polynomial of degree ( = 3; 4; 5) with
, , ≥ 0. Then ( ; ; ) ≥ 0 ⇔ ( ; 1; 1) ≥ 0 and (0; ; ) ≥ 0.
5) (S.O.S technique) Define = ( − ) + ( − ) + ( − ) , where ; ; are
functions with variables ; ; . Then ≥ 0 iff:
* ; ; ≥0
* ≥ ≥ ; ≥ 0; + ≥ 0; + ≥ 0
* ≥ ≥ ; ≥ 0; ≥ 0; + 2 ≥ 0; + 2 ≥ 0
* ≥ ≥ ; ≥ 0; ≥ 0; + ≥0
* + + ≥ 0; + + ≥0
→ Consider ( ; ; ) = ( − ) + ( − )( − ) ≥ 0 (*)
* If ( ; ; ) is symmetric then to prove (*) is true, we assume that ≥ ≥ or
= min{ ; ; } or = max{ ; ; } and prove that , ≥ 0.
* If ( ; ; ) is cyclic then to prove (*) is true, we assume that = min{ ; ; } or
= max{ ; ; } and prove that , ≥ 0. (S.S technique)
c) Some identities:
(*) Some useful identities in inequality that can be proved by S.O.S, S.S technique:
( )
1) + −2 =( − ) ; + −2=
∑ −
→ ( + + ⋯+ )−( + +⋯+ )=
( + + ⋯+ )+( + + ⋯+ )
( )
For = 2: 2( + )−( + )=
( )
( ) ( ) ( ) ( ) ( )( )
For = 3: 3( + + )−( + + )= =2
( ) ( ) ( ) ( )
1 1 1 ( − ) ( − ) ( − ) 2( − ) 1 1
→( + + ) + + −9 = + + = + + ( − )( − )
2) ( + + ) − 3( + + ) = + + −( + + )=
1
= [( − ) + ( − ) + ( − ) ] = ( − ) + ( − )( − )
2
3) + − ( + ) = ( + )( − )

12 ROMANIAN MATHEMATICAL MAGAZINE NR. 22


Romanian Mathematical Society-Mehedinți Branch 2019
3 ( − ) ( − ) ( − )
→ + + = − + + =
+ + + 2 2( + )( + ) 2( + )( + ) 2( + )( + )
( − ) ( + + 2 )( − )( − )
= +
( + )( + ) 2( + )( + )( + )
→ 3( + + ) − ( + + )( + + ) = ( + )( − ) + ( + )( − ) + ( + )( − )
= 2( + )( − ) + ( + + 2 )( − )( − )
4) ( + + )( + + ) − 9 = ( + )( + )( + ) − 8 =
= ( − ) + ( − ) + ( − ) = 2 ( − ) + ( + )( − )( − )
5) + + − 3 = ( + + )[( − ) + ( − ) + ( − ) ] =
= ( + + )[( − ) + ( − )( − )]
6) + + − 3 = [( − ) (3 + − ) + ( − ) (3 + − ) + ( − ) (3 + − )] =
( − ) ( − )( − )
= +
( ) ( ) ( )
7) + + −( + + )= + + = + ( − ) + ( − )( − )
8) + + +3 − ( + )− ( + )− ( + ) =
1
= [( + − )( − ) + ( + − )( − ) + ( + − )( − ) ]
2
= ( + − )( − ) + ( − )( − )
9) + + −( + + )=
1
= {( − ) [( + ) + ] + ( − ) [( + ) + ] + ( − ) [( + ) + ]}
2
( + ) + 1 ( + )( + )
= ( − ) + + ( − )( − )

10) + −( + + ) = + ( − ) + + ( − ) + + ( − )
+
11) ++ + + + − ( + )− ( + )− ( + )=
1
= [( + )( − ) + ( + )( − ) + ( + )( − ) ]
2
= ( + )( − ) + ( + + + )( − )( − )
(**) More identities: Actually the first nine identities are rare, so just consider the identity
10 onwards:
1) * ⋅ + ⋅ + ⋅ = + + − 1 with ≠ ≠
* ⋅ + ⋅ + ⋅ = − − − − 1 with ≠ ≠
( )( )( )
2) * + + =( )( )( )
( ) ( ) ( )
* + + = ( )( )( )
with ≠ ≠
3) * ⋅ + ⋅ + ⋅ = −1 with ≠ ≠
( ) ( ) ( )
* ⋅ + ⋅ + ⋅ = ( )( )( )

4) * ⋅ + ⋅ + ⋅ = + +
* ⋅ + ⋅ + ⋅ = −( + + ) with ≠ ≠
5) ( − )( + ) + ( − )( + ) + ( − )( + ) = 0
( + )( − ) + ( + )( − ) + ( + )( − ) = −2( − )( − )( − )

13 ROMANIAN MATHEMATICAL MAGAZINE NR. 22


Romanian Mathematical Society-Mehedinți Branch 2019
( ) ( ) ( )
6) ( )( )
+( )( )
+( )( )
= 1 with ≠ ≠
7) ⋅ + ⋅ + ⋅ = −1 with ≠ ≠
8) * ⋅ + ⋅ + ⋅ = + + with ≠ ≠
* ⋅ + ⋅ + ⋅ = −( + + )[ ( − ) + ( − ) + ( − ) ]

9) ⋅ + ⋅ + ⋅
= −1
10) ( + + ) = + + + 3( + )( + )( + )
11) 2( + + )−( + + ) = ( + + )( + − )( + − )( + − )
12) ( + + )−( + + ) = ( − )( − )( − ) ∑ with
, , ∈ and ≥ 2 such that + + = − 2 (?)
E.g: for = 2: ( + + )−( + + ) = ( − )( − )( − )
= 3: ( + + )−( + + ) = ( − )( − )( − )( + + )
= 4: ( + + )−( + + ) = ( − )( − )( − )( + + + + + )
13) ( − )( − ) + ( − )( − ) + ( − )( − ) =
1
= [( + − )( − ) + ( + − )( − ) + ( + − )( − ) ]
2
14) If , , ≠ 0 such that = 1 then: + + =1
15) + =4⇔2 + + = (4 − )(4 − ), etc ⇔
+
2 + 2 + 2 +
⇔ + + =
(2 + )(2 + ) (2 + )(2 + ) (2 + )(2 + )
(2 − )(2 − ) (2 − )(2 − ) (2 − )(2 − )
= + + = + + =
2 + 2 + 2 + 2 + 2 + 2 +
1 1 1 1
= + + =1⇔ + + = =
2 + 2 + 2 + 2 + 2 + 2 + + + −2
+ + +2
= ⇔ ( + + − 2) = (2 − )(2 − )(2 − )
2 ( + + )−
→ From the identity, there exists , , > 0 such that: =2 ( )( )
; =2 ( )( )
;

=2 ( )( )
. And there exists triangle such that: = 2 cos ; = 2 cos ;
= 2 cos
16) + + +2 =1⇔ + = (1 − )(1 − ), etc ⇔
+ + +
⇔ + + =
(1 + )(1 + ) (1 + )(1 + ) (1 + )(1 + )
(1 − )(1 − ) (1 − )(1 − ) (1 − )(1 − )
= + + =1⇔
+ + +
⇔ + =2⇔ ++ + =1⇔
+ + + + + +
1 1 1 2 + + +1
⇔ + + = = ⇔
+ + +
+ + −1 + + −
⇔ ( + + − 1) = 2(1 − )(1 − )(1 − )
→ If we substitute → ; → ; → we will get identity 15, so:

From the identity, there exists , , > 0 such that: = ( )( )


; = ( )( )
;

14 ROMANIAN MATHEMATICAL MAGAZINE NR. 22


Romanian Mathematical Society-Mehedinți Branch 2019

= ( )( )
. And there exists triangle such that: = cos ; = cos ; = cos .
⨁ Also if we let = ; = ; = with , , > 0 then = , so we have 2
identities 17-18:
17) + + + =4⇔ + + = + + =1⇔
√ √
⇔ + + =
2+ 2+ 2+ + +√ −2
18) + + +2 =1⇔ + + =2⇔ + + =1⇔
√ √ 2
⇔ + + =
1+ 1+ 1+ + +√ −1
→ From identity 17, there also exists, , , > 0 such that = ; = ; = ,
similarly for identity 18.
19) ( + + )+ ( + + )+ ( + + )= ( + + )+
+ ( + + ) + ( + + ) = ( + + )( + + )
( + + ) + ( + + ) + ( + + ) (
= + + )( + + )
20) ( + + )( + + )( + + ) = + + , where
= + + ; = + +
= 3( + + )( + + ) + [( − )( − )( − )]
3 1
= [ ( + ) + ( + ) + ( + )] + [( − )( − )( − )]
4 4
1
= [( + + ) ( + + )+( + + ) ( + + )]
2
∗( + + ) ( + + ) = ( + 2 )( + 2 )( + 2 ) + [( − )( − )( − )]

∗( + + ) ( + + ) = (2 + )(2 + )(2 + ) + [( − )( − )( − )]
21) 2( + )( + )( ) =[ ( + )+ ( + )+ ( + )−2
+ ] +
+[( − )( − )( − )]
22) 2[ ( − ) + ( − ) + ( − ) ]=[ ( − ) + ( − ) + ( − ) ] +
+[( − )( − )( − )]
23) + + = ⇔ = ⇔ = or = or =
d) Useful lemmas: In above parts, I’ve showed some of it. In this part I will state more
lemmas, maybe a lot but worth it :)
• Inequalities with condition about , , (denote = + + ; = + + ;
= ) – part 1: In this part, I will state lemma with familiar conditions, about the
„unusual” conditions, I will show in later seasons.
d. 1) If , , > 0 such that = then:
1) + + ≥ + + ( , ∈ ; > )
2) + + ≥ + + ; + + ≥ + + → + + ≥ ( + + − 1) →
1 1 1
→ + + +3≥ + + + + +
3) + + ≥ 1 with ∈
4) + + ≤ 1 with ∈

15 ROMANIAN MATHEMATICAL MAGAZINE NR. 22


Romanian Mathematical Society-Mehedinți Branch 2019

5) * ( )
+ ( )
+ ( )

∗ ( )
+ ( )
+ ( )
≥ → ( )
+ ( )
+ ( )
≥ with , > 0 (*)
6) + + ≤
7) + + ≤1
8) Let = + + ; = + + then:
+ ≥4
* +3≥4 → ∗ ≥5 −6
+ ≥ 4⋅

9) + + ≥1→ + + ≥ + + ≥1≥
1 1 1 1 1 1 1 1 1
≥ + + → + + ≥1≥ + +
+2 +2 +2 2 +1 2 +1 2 +1 +2 +2 +2
1 1 1 3 1 1 1
: + + ≥ ≥ + +
3 +1 3 +1 3 +1 4 +3 +3 +3
10) ( + 1)( + 1)( + 1) ≥ ( + )( + )( + ) ≥ ( + 1)( + 1)( + 1) ≥ 8
11) ( − 1) + 1 + ( − 1) + 1 + ( − 1) + 1 ≤ ( + + ) with
+ + ≥ + +
→ Special case: + + = + + and without condition = 1.
12) ( )
+( )
+( )

13) ( + + ) ≥ 81( + + )
d. 2) If , , > 0 such that + + = then:
1) + + ≥ 3 ≥ + +
2) + + ≥ 3 for ∈
3) + + ≥ 3 for , ∈ such that ≥
4) + + ≥ + +
5) + + ≤ + + ≤ + + with all ∈
6) + + ≤ with all ≥2
7) + + ≥ with , ∈ such that >
8) ( + + )≤3→( ) ⋅( + + ) ≤ 3 with ∈ → + + ≥
1 1 1
≥ + + ≥ + +
9) + +≥1
10) + + + ≤4
11) + + ≥ + +
12*) + + ≥ with ≥1
d.3) If , , > 0: + + = 3 then:
1) + + ≥ 3 → ( + + ) ≥ 243
2) + + ≥ + +
3) + + ≥ →( )
+( )
+( )

16 ROMANIAN MATHEMATICAL MAGAZINE NR. 22


Romanian Mathematical Society-Mehedinți Branch 2019
4) + + ≤ + +
5) ( )
+( )
+( )
≤ with ≥2
→( )
+( )
+( )
≤( )
with , > 0 and
≥ 2( − + ) (?)
6) + + ≤ with ≥1
7) + + ≤1
8*) + + ≤1
d.4) If , , > 0 such that + + = then:
1) + + ≥ + + → √ +√ +√ ≥ + + with ∈
2) + + ≥ → ( + + ) ≥ 9( + + )
3) + + ≤ 1; + + ≥
4) + + ≤3
5) + + ≤ 2+ → In case , , ≥ 0: the equality happens
= = =1
if
= √2; = 1; = 0
d.5) If , , > 0 such that + + = + + then:
1) + + ≤1
∗ + + +1≥4
2) + + + ≥4→
∗ ( + 1)( + 1)( + 1) ≥ 8
3) + + ≤
4) ( + )( + )( + ) ≥ 8
5) + + ≥ + + → + + + ≥ 3+ with ≤3
6) + + ≥ +2
d. 6) If , , > 0 such that + + + = then:
1) + + ≥ + + ≥3→ + + ≥

→ + + ≥ + +
2) ≥ + + ≥ → ≥ + +
3) + + ≤1
4) √ + √ + √ ≤ 3
5) + + ≥ + +
6) + + ≤ + +
d.7) If , , > 0 such that + + + = then:
1) + + ≤ + + ≤3→ + + ≤ +2 ≤ + + ≤3
2) ≤ + + ≤ ( )
3) ( + ) + ( + ) + ( + )≤6
4) + + ≥ 3 → + + ≥ + +
5) + ≤ + ≤ 2; etc

17 ROMANIAN MATHEMATICAL MAGAZINE NR. 22


Romanian Mathematical Society-Mehedinți Branch 2019

6) + + + + + ≥ 2( + + )→ + + ≥ √ +√ +√ ≥ + +
d.8) If , , > 0 such that + + = ( + + ) then: ≥√
→ the equality happens if ( ; ; )~( ; ; ) with > 0.
d.9) If , , > 0 such that + + = + then:
1) Assume that ( − 1)( − 1) ≥ 0 then: + ≥2→ + + + ≥4
2) max{ ; ; } ≥ 1; max{ ; ; } ≥ 1
• Inequalities with classic condition (like , , > 0, …) – part 1:
d.10) If , , > 0 then: + + ≥

d.11) If ; ;… ; > 0 then: + + ⋯+ ≥

d.12) If , , ≥ 0, no 2 of which are 0 then: + + ≤3⋅ with


∀ ∈
d.13) If , , > 0 then: + ≥ + + ≥ ( )
+1
d.14) If , , , , , > 0 then:
( + )+ ( + )+ ( + )≥2 ( + + )( + + )
d.15) If , , , ≥ 0 then:
+ + + +2 ≥ + + + + + (Turkevich’s
inequality)→ The equality happens iff = = = and = = = > 0, = 0 or any
cyclic permutation.
d.16) If , , > 0 then: + + ≥4 + +
d.17) If , , are sides of a triangle then:
3( + + )≥ √ +√ +√ ≥√ + − +√ + − +√ + −
d.18,19) If , , > 0 then:
* + + ≥ ≥
( )
* + + ≥ + + ≥
d.20) If , , > 0 then:
+ +
+ + ≥ ≥ + +
+ + + + + + + + +2 +2 +2
d.21) If , , > 0 then: + + ≥ 3
d.22) If , , , , > 0 then: + + ≥

d.23*) If ≥ 0 then: ≥1+ + ≥ 1+ → ≥ 1+



d.24) If > 0 ∀ = 1; 2; … ; then: ∑ ≥ ( = )

d.25) If , , > 0 then: + + +( )


≥2
d.26) If , , > 0 then:
+ + +2 + 1 ≥ 2( + + )→ + + + + 5 ≥ 3( + + )
d.27*) If , , are sides of a triangle then: ( − )+ ( − )+ ( − )≥0
(IMO 1983)
d.28) If , , > 0 and ∈ then: + + ≥ + + ≥⋯≥

18 ROMANIAN MATHEMATICAL MAGAZINE NR. 22


Romanian Mathematical Society-Mehedinți Branch 2019

≥ + + ≥ + + ≥ + +
d.29) If , , > 0 then: + + ≤1
( )( ) ( )( ) ( )( )

d.30) If , , > 0 then: + + ≤



d.31) If , , are sides of a triangle then: + + ≤ 2( + + ) (the
equality happens iff , , are sides of a degenerate triangle.)
d.32*) If , , > 0 and ≥ 0 then: + + ≥ + +
d.33) If , , > 0 then: + + + ( )( )( )
≥4
d.34) If , , ≥ 0, no 2 of which are 0 then: +( )( )( )
≥ 2 (Jack
Garfunkel’s inequality)
• Inequalities with classic condition – part 2: In this part I will show some inequalities about
variables , , (denote = + + ; = + + ; = ), even Schur’s inequality.
Firstly, we have some identities about , , :
1) + + = −2
2) + + = −2
3) ( + )( + )( + ) = − → ( + )+ ( + )+ ( + ) = −3
(
4) + )( )
+ + + ( )( )
+ + + ( )( + = )
5) + + = −3 +3
6) + + = −3 +3
7) + + = −4 +2 +4
8) ( + ) + ( + ) + ( + ) = −2 −
9) Denote = −4 + + 18 − 4 − 27 , then:
* [( − )( − )( − )] =
√ √
* + + = if ( − )( − )( − ) < 0; if
( − )( − )( − ) ≥ 0
√ √
* + + = if ( − )( − )( − ) < 0; if
( − )( − )( − ) ≥ 0
10) ( + )+ ( + )+ ( + ) = −2 + +4 −2 −3
11) ( + ) + ( + ) + ( + ) = − −4 +7 +2 −3
12) + + = −6 +6 +9 − 12 −2 +3
• Some inequalities about the relation of , , :
1) ≥3 ; ≥3
2) ≥9
27 9
→ ≥ ( − )≥3 ≥ ≥ 27
8
3) +9 ≥4 → ≥ → ≥ max 0;
4) 2 + 9 ≥ 7
5) +3 ≥4
6) + 4 + 6 ≥ 5
7) ≥ → ≥ max 0;

19 ROMANIAN MATHEMATICAL MAGAZINE NR. 22


Romanian Mathematical Society-Mehedinți Branch 2019

8) ≤ ; ≤ . Combining inequality 3, 6, 8 and we get:


(5 − ) −7 + 13 (4 − ) (4 − )( − )
min ; ≥ ≥ max 0; ;
18 9 9 6
√ √
9*) ≤ ≤ , with = − 3 . This result comes from solving the
inequality = −4 + + 18 − 4 − 27 ≥ 0 with variable . → From this
result, by AM-GM we have:
( )
* 27 ≥ 9 −2 −2 √ =9 −2 − ( )
(9 − 2 )( −2 )−(
− 3 )[( − 2 ) + ( − 3 )]
≥ =
( −2 )
( − 2 )(−3 + 14 −6 )− ( −3 ) (4 − )(4 − 10 +3 )
= =
( −2 ) ( −2 )
( )
* 27 ≤ 9 −2 +2 √ =9 −2 +

(9 −2 )(2 − 3 ) + 2( −3 ) − + ( −3 )
≤ =
(2 − 3 )
(2 − 3 )[2(9 − 2 ) + ( − 3 )(2 − 3 )] + 4 ( − 3 )
=
2 (2 − 3 )
(2 − 3 )(−2 + 9 +9 )+4 ( −6 + 9 ) 27 ( − )
= =
2 (2 − 3 ) 2 (2 − 3 )
So we have the chain: ≥ ( )
≥ ≥ ( )
, more interesting, the
third inequality is stronger than Schur deg 3 and 4, since:

( )
− = ( )
≥ 0 in case 4 ≥
(4 − )(4 − 10 +3 ) (4 − )( − ) (4 − ) ( − 3 )
− = ≥0
27 ( − 2 ) 6 54 ( − 2 )
Hence we can also conclude that: ≥ max 0; ( )
√ √
Further more, we can write inequality 9 as: ≤ ≤
+√ − 2√ −√ + 2√
⇔ ≤ ≤
27 27
For my thinking, the , , technique is one of the nicest and hardest techniques to use, due
to long calculations, time loss and it requires carefulness in calculation. But the nice thing
to say is its benefit in solving hard inequality about symmetric and cyclic expressions.
There is a similar technique to , , , that is , , technique (can be found and search in
inequality sites and forums)
ELEMENTE DE GEOMETRIA TRIUNGHIULUI IN COORDONATE BARICENTRICE

By Neculai Stanciu-Romania
Articolul vine uşor în completarea programei şcolare din liceu şi are scopul de a pune în
evidenţã noi metode de rezolvare a problemelor de geometrie şi de a lãrgi orizontul

20 ROMANIAN MATHEMATICAL MAGAZINE NR. 22


Romanian Mathematical Society-Mehedinți Branch 2019
matematic al elevilor.În cele ce urmeazã, voi enunţa şase teoreme importante şi voi
demonstra numeroase aplicaţii ale acestor teoreme referitoare la unele egalitãţi şi inegalitãţi
în triunghi.
Teorema 1.Se considerã un triunghi fix ABC şi notãm BC=a,CA=b,AB=c,S=Aria(ABC).
Atunci pentru orice M  E2 (unde, E2 este planul euclidian)existã şi este unic tripletul ordonat
(x,y,z)  R3,x+y+z=1 astfel încât x MA  y MB  z MC  0
şi reciproc,pentru orice triplet ordonat (x,y,z)  R3,x+y+z=1 existã şi este unic un punct M  E2
astfel încât x MA  y MB  z MC  0 şi în acest caz vom spune cã punctul M are coordonatele
baricentrice (x,y,z) în raport cu triunghiul ABC şi vom nota M(x,y,z).Pentru orice X  E2 avem
x XA  y XB  z XC  XM (demonstraţie în [1]pag.66).
Exemple de coordonate baricentrice pentru câteva puncte remarcabile într-un triunghi:
1.1.A(1,0,0),B(0,1,0),C(0,0,1);
1 1 1
1.2.G , ,   centrul de greutate;
3 3 3
 a b c 
1.3.I  , ,   centrul cercului inscris C ( I , r );
 2p 2p 2p 
 a b c 
1.4.I a  , ,   centrul cercului exinscris C ( I a , ra );
 2( p  a ) 2( p  a ) 2( p  a ) 
 pa pb pc
1.5.N  , ,   punctul lui Nagel;
 p p p 
 ( p  b)( p  c) ( p  c)( p  a ) ( p  a )( p  b ) 
1.6. , ,   punctul lui Gergone;
 r (4 R  r ) r (4 R  r ) r (4 R  r ) 
1.7.H ctgBctgc, ctgCctgA, ctgActgB  ortocentrul ;
R 2 sin 2 A R 2 sin 2 B R 2 sin 2C
1.8.O( , , )  centrul cercului circumscris C (O, R);
2S 2S 2S
a2 b2 c2
1.9.L ( 2 , , )  punctul lui Lemoine.
a  b2  c2 a2  b2  c2 a2  b2  c2
Teorema 2.Puterea punctuluiM(x,y,z)  E2 faţã de cercul C(O,R),circumscris triunghiului
ABC este dat de relaţia:
pc ( M )  ( yza 2  zxb 2  xyc 2 ); OM 2  R 2  pc M .
(demonstraţie în [1] pag.68).
Aplicaţii ale teoremei 2.
a2  b2  c2 a2  b2  c2
2.1. p c (G )   , OG 2  R 2  si,
9 9
a 2  b 2  c 2  9R 2 ;
Demonstraţie:se aplicã teorema2 şi 1.2.
2.2. pc ( I )  2 Rr , OI 2  R 2  2 Rr , si, R  2r ;
Demonstraţie:rezultã imediat din teorema2 şi 1.3.
2
2.3. pc ( I a )  2 Rra , OI a  R 2  2Rra ;

21 ROMANIAN MATHEMATICAL MAGAZINE NR. 22


Romanian Mathematical Society-Mehedinți Branch 2019
Demonstraţie:folosim teorema2 şi 1.4.
2.4. pc ( N )  4r ( R  r ), ON  R  2r ;
Demonstraţie:utilizãm teorema 2 şi 1.5.
2.5. p c ( H )  8 R 2 cos A cos B cos C , OH 2  R 2 (1  8 cos A cos B cos C ),
1
si, cos A cos B cos C  , iar
8
OH 2  9 R 2  (a 2  b 2  c 2 ), deoarece, OH  3OG.
Demonstraţie:se aplicã teorema 2 şi 1.7.
 2p 
2.6. pc ()   r ( R  r ) ;
 4R  r 
Demonstraţie:se foloseşte teorema 2 şi 1.6.
2 2
 abc   abc 
2.7. pc ( L)  3 2 2 2 
, OL2  R 2  3 2 2 2 
, si,
a b c  a b c 
a 2  b 2  c 2  4 S 3;
Demonstraţie:rezultã imediat din teorema 2 şi 1.9.
Teorema 3.a)Pentru M(x,y,z)  E2 existã relaţia:
xMA 2  yMB 2  zMC 2   pc ( M )
b)Pentru orice X E2 existã relaţia:
xXA  yXB 2  zXC 2  XM 2  pc ( M ), si
2

xXA 2  yXB 2  zXC 2  yza 2  zxb 2  xyc 2 .


(demonstraţie în [1] pag.69)
Aplicaţii ale teoremei 3 :
a2  b2  c2
3.1.M  G  XA 2  XB 2  XC 2  3 XG 2  X  E 2 ;
3
a2  b2  c2
GA 2  GB 2  GC 2  ;
3
a2  b2  c2
XA 2  XB 2  XC 2  X  E 2 ,
3
cu egalitate pentruX  G.
3.2.M  I  aXA 2  bXB 2  cXC 2  2 pXI 2  abc, X  E 2 ;
aIA2  bIB 2  cIC 2  abc; aXA2  bXB 2  cXC 2  abc, X  E 2 ,
cu egalitate pentruX  I .
2
3.3.M  I a  bXB 2  cXC 2  abc  aXA 2  2( p  a ) XI a , X  E 2 ;
X  I a  bI a B 2  cI a C 2  abc  aI a A 2 ; bXB 2  cXC 2  abc  aXA 2
cu egalitate pentruX  I a .
3.4.DacaM ( x, y, z )  E 2 , atunciX  E 2 avem :
yza 2  zxb 2  xyc 2  xXA 2  yXB 2  zXC 2  XM 2  R 2 , cu egalitate in s tan ga, pentruX  M
si egalitate in dreapta, pentruM  O.De exemplu, pentruM  I 
abc  aXA 2  bXB 2  cXC 2  2 p ( XI 2  R 2 ).

22 ROMANIAN MATHEMATICAL MAGAZINE NR. 22


Romanian Mathematical Society-Mehedinți Branch 2019

3.5.Daca, X  M  xMA2  yMB 2  zMC 2 


 yza 2  zxb 2  xyc 2  R 2  OM 2 , si xMA2  yMB 2  zMC 2  R 2 , cu egalitate pentruM  O;
3.6.Daca, X  A  MA 2  p c ( M )  zb 2  yc 2 , de unde, rezulta :
MA 2  MB 2  MC 2  3 p c (M )  ( y  z )a 2  ( z  x)b 2  ( x  y )c 2 .
Teorema 4.Dacã
M k ( xk , yk , z k )  E2 , k  1,2 ,atunci distanţa între punctele M ,M este datã de relaţia:
1 2
2
 
M 1 M 2   ( y1  y 2 )( z1  z 2 ) a  ( z1  z 2 )( x1  x 2 )b  ( x1  x 2 )( y1  y 2 ) c 2 .
2 2

(demonstraţie în [1] pag.70).


Aplicaţii ale teoremei 4: Utilizând coordonatele baricentrice (vezi exemplele date)şi
teorema 4 obţinem urmãtoarele relaţii:
a2  b2  c2
4.1.OG 2  R 2  ;4.2.OH 2  R 2 (1  8 cos A cos B cos C ) 
9
9 R  (a  b  c )  a 2  b 2  c 2  8 R 2 (1  cos A cos B cos C ), de unde rezulta,
2 2 2 2

a 2  b 2  c 2  8 R 2  ABCeste, dreptunghic
4.3.OI 2  R 2  2 Rr; HN  2OI ; 4.4.NI 2  9GI 2  p 2  5r 2  16 Rr 
p 2  5r 2  16 Rr; 4.5. HI 2  4 R( R  r )  3r 2  p 2  4 R( R  r )  3r 2 ;
  p 3 2  2
2 2
4.6.I  r 1      p 3  4 R  r ; 4.7.LI 2  42S R(2R  r ) ;
  4 R  r   p  r  4 Rr
 
4
4.8.G 2 
9( 4 R  r ) 2
 
p 2 (4 R 2  8Rr  5r 2 )  r (4 R  r ) 3  ( 2 R  r )(2 R  5r ) p 2  r (4 R  r ) 3 ;

 2 p 2 (2 R  r ) 
4.9.H 2  4 R 2 1  2 
 2 p 2 (2 R  r )  R ( 4 R  r ) 2 .
 R (4 R  r ) 
M ( x , y , z )  E2 , k  1,2
Teorema 5. Dacã k k k k ,atunci avem:
1

OM 1 OM 2  R 2  ( y1 z 2  y2 z1 ) a 2  ( z1 x2  z 2 x1 )b 2  ( x1 y2  x2 y1 )c 2 .
2

(demonstraţie în [1] pag.71.).Aplicaţii ale teoremei 5: Folosind aceastã teoremã obţinem
egalitãţi şi inegalitãţi importante printre care cele ce urmeazã:
1
5 .1 .OG OM  R 2 
6
 
( y  z ) a 2  ( z  x )b 2  ( x  y ) c 2

1
5 .1 .1 .OG OA  R 2  (b 2  c 2 ); OG  OA  b 2  c 2  6 R 2 , si , b 2  c 2  6 R 2
6
0
 m ( AOG )  90
1
5 .1 .2 .OG OI  R 2  ( p 2  r 2  2 Rr ), si , 2 R (3 R  r )  p 2  r 2  m ( IOG )  90 0 , iar ,
6
2 R (3 R  r )  p 2  r 2  a  b  c , sau , OI  OG ;
p2
5 .1 .3 .OG ON  ( R  r ) 2  , si , p  3 ( R  r )  m ( NOG )  90 0 , iar ,
3
p  3 ( R  r )  a  b  c , sau , ON  OG ;

23 ROMANIAN MATHEMATICAL MAGAZINE NR. 22


Romanian Mathematical Society-Mehedinți Branch 2019

1
5.2.OI OM  R( R  r )  ( xbc  yca  zab );
2
1
5.2.1.OI ON  R 2  5 Rr  ( p 2  r 2 ), si ,2 R( R  5r )  p 2  r 2
2
0
 m( ION )  90 , iar ,
2 R( R  5r )  p 2  r 2  a  b  c, sau, OI  ON ;
1
5.2.2.OI OA  R 2  Rr  bc, si, aR  (b  c)r  m( AOI )  90 0 , iar ,
2
aR  (b  c)r  a  b  c, sau , OA  OI .
Teorema6. Dacã
M k ( x k , y k , z k )  E 2 , k  1,2, avem :
1
a) IM 1 IM 2  4 Rr  ( z1  z 2 )ab  ( x1  x 2 )bc  ( y1  y 2 )ca  
2
1

2
 
( y1 z 2  y 2 z1 )a 2  ( z1 x 2  z 2 x1 )b 2  ( x1 y 2  x2 y1 )c 2 ;
(demonstraţie în [1],pag.73)
2 2 1
b )GM 1 GM 2 
9 6
 
(a  b 2  c 2 )  ( x1  x2 )a 2  ( y1  y2 )b 2  ( z1  z2 )c 2 

1

2

( y1 z2  y2 z1 )a 2  ( z1 x2  z2 x1 )b 2  ( x1 y2  x2 y1 )c 2 ; 
(demonstraţie în [1],pag.74)
1
c) MM 1 MM 2  pc ( M )   y( z1  z 2 )  z ( y1  y2 )  ( y1 z 2  y2 z1 ) a 2  ....
2
(demonstraţie în [1],pag.75)
Observaţie.Cu ajutorul acestor relaţii remarcabile se pot determina ,în particular,produse
scalare,distanţe,egalitãţi şi inegalitãţi utilizând puncte din mulţimea:
A, B, C , O, G, H , I , I a , I b , I c , N , , L
asociatã unui triunghi ABC.(Exerciţiu!).
Mai fac observaţia cã particularizãri şi unele extinderi ale coordonatelor baricentrice sunt
abordate în lucrãrile [2] , [3] şi articolele din [4] şi [5] .Un fapt care motiveazã studiul
coordonatelor baricentrice este legãtura acestora cu calculul vectorial recent (relativ)
introdus în programele şcolare IX-XII .
Bibliografie
[1] .V.Nicula,Geometrie planã,Ed.Gil,2002.
[2].N.Teodorescu,ş.a.,Culegere de probleme pentru concursurile de matematicã,
vol.5, S.S.M.R, Bucureşti,1977.
[3]. M.Craioveanu,I.D.Albu,Geometrie afinã şi euclidianã,Ed.Facla,Timişoara,1982.
[4] .T. Bârsan, Recreaţii matematice, nr. 1 / 2002;
[5] .C. Coandã, Gazeta matematicã, nr. 8 / 2005;
[6].N.Stanciu, Matematicǎ gimnaziu & liceu, Editura ’’Rafet’’, Rm. Sãrat, 2007

24 ROMANIAN MATHEMATICAL MAGAZINE NR. 22


Romanian Mathematical Society-Mehedinți Branch 2019

FEW APPLICATIONS OF COCEA’S INEQUALITY


By Daniel Sitaru, Claudia Nănuți-Romania
Abstract: In this paper are developed a few applications of a 1993’s inequality of Romanian
mathematician C. Cocea.
Keywords: circumcentre, centroid, orthocentre, incentre, Nagel, Lemoine
Cocea’s inequality
Let be a point in ( ), where , , necollinears. If = ; = ; = then:
+ + ≥ (*)
Proof:
If is the origin of the system of rectangular axis lets denote , , the affixes of ,
respectively . ( ); ( ); ( ); , , different in pairs

1 = | 1| = + + ≤
( − )( − ) ( − )( − ) ( − )( − )

≤ + + =
( − )( − ) ( − )( − ) ( − )( − )
| |⋅| | | |⋅| | | |⋅| |
= + +
| − |⋅| − | | − |⋅| − | | − |⋅| − |
| − |⋅| |⋅| |+| − |⋅| |⋅| |+ | − |⋅| |⋅| | ≥ | − |⋅| − |⋅| − |
+ + ≥
Application 1
In Δ the following relationship holds:
≥ 2 (Euler’s inequality)
Solution:
Let = 0 (circumcenter) in (*)
= = = = = =
⋅ ⋅ + ⋅ ⋅ + ⋅ ⋅ ≥
( + + )≥4

25 ROMANIAN MATHEMATICAL MAGAZINE NR. 22


Romanian Mathematical Society-Mehedinți Branch 2019
⋅2 ≥4 ⇒2 ≥4 ⇒ ≥2
Application 2
In Δ the following relationship holds:
4( + + )≥9
Solution:
Let = (centroid) in (*)
2 2 2
= ; = ; =
3 3 3
2 2 2 2 2 2
⋅ ⋅ + ⋅ ⋅ + ⋅ ⋅ ≥
3 3 3 3 3 3
4( + + )≥9
Application 3
In Δ the following relationship holds:
sin sin sin 2
+ + ≥
sin sin sin sin sin sin

Solution:
Let = (incentre) in (*)

= ; = ; =
sin sin sin

⋅ ⋅ + ⋅ ⋅ + ⋅ ⋅ ≥
sin sin sin sin sin sin

+ + ≥
sin sin sin sin sin sin
2 sin 2 sin 2 sin 4
+ + ≥
sin sin sin sin sin sin
sin sin sin 4 2
+ + ≥ =
sin sin sin sin sin sin ⋅2

Application 4
In Δ the following relationship holds:

sin |cos cos | + sin |cos cos | + sin |cos cos | ≥


2
Solution:
Let = (orthocenter) in (*)
26 ROMANIAN MATHEMATICAL MAGAZINE NR. 22
Romanian Mathematical Society-Mehedinți Branch 2019
= 2 |cos |; = 2 |cos |; = 2 |cos |
⋅ 2 |cos | ⋅ 2 |cos | + ⋅ 2 |cos | ⋅ 2 |cos | + ⋅ 2 |cos | ⋅ 2 |cos | ≥

|cos cos | + |cos cos | + |cos cos | ≥


4
4
2 sin ⋅ |cos cos | + 2 sin |cos cos | + 2 sin |cos cos | ≥ =
4
sin ⋅ |cos cos | + sin |cos cos | + sin |cos cos | ≥
2
Applicaton 5
In Δ the following relationship holds:

(( − ) + 4 )(( − ) + 4 ) ≥
( , , )

Solution:
Let = (Nagel’s point) in (*)

= ( − ) +4 ; = ( − ) +4 ; = ( − ) +4

⋅ ⋅ ≥
( , , )

(( − ) + 4 )(( − ) + 4 ) ≥
( , , )

Application 6
In Δ the following relationship holds:
( + + )
+ + ≥
4
Solution:
Let = (Lemoine’s point) in (*)
2 2 2
= ; = ; =
+ + + + + +
2 2
⋅ ⋅ ≥
+ + + +
( , , )

4 ≥
( + + )
( , , )

( + + )
+ + ≥
4
In all applications equality holds if = = (equilateral triangle)
27 ROMANIAN MATHEMATICAL MAGAZINE NR. 22
Romanian Mathematical Society-Mehedinți Branch 2019
Refferences:
[1] M. Becheanu - “Mathematical Olympiads” – “Gil” Publishing House – Zalău -1997
[2] Mihály Bencze; Daniel Sitaru – “699 Olympic Mathematical Challenges” – “Studis”
Publishing House – Iași – 2017
[3] Mihály Bencze; Daniel Sitaru – “Quantum Mathematical Power” – “Studis” Publishing
House – Iași – 2017
[4] Mihály Bencze; Daniel Sitaru – “Olympic Mathematical Energy” – “Studis” Publishing
House – Iași – 2018
[5] George Apostolopoulos; Daniel Sitaru – “The Olympic Mathematical Marathon” –
“Cartea Românească” Publishing House – Pitești – 2017
[6] Romanian Mathematical Magazine – Interactive Journal – www.ssmrmh.ro

ABOUT PROBLEM UP.147.


ROMANIAN MATHEMATICAL MAGAZINE-AUTUMN EDITION-2018

By Marin Chirciu-Romania
1) In  ABC
ra2 rb2 rc2 9 a2  b2  c2 
   .
B C C A A B 4
tg tg tg tg tg tg
2 2 2 2 2 2
Proposed by Hoang Le Nhat Tung, Hanoi, Vietnam
Soluție:
Demonstrăm rezultatul ajutător:
Lemă.
2) In  ABC
3 2
ra2

rb2

rc2

 4 R  r   12 Rp .
B C C A A B r
tg tg tg tg tg tg
2 2 2 2 2 2
Demonstrație.
S B C pa
Folosind ra  și tg tg  obținem:
pa 2 2 p

28 ROMANIAN MATHEMATICAL MAGAZINE NR. 22


Romanian Mathematical Society-Mehedinți Branch 2019

S2
2 3 3
ra2  p  a 2 1 2  4 R  r   12 Rp 2  4 R  r   12 Rp 2
 B C  p  a S p  p  a 3  S p  S 3

r
.
tg tg
2 2 p
Să trecem la rezolvarea inegalității din enunț.
Folosim Lema și a 2  b 2  c 2  2  p 2  r 2  4 Rr  inegalitatea se scrie:
3
 4R  r   12 Rp 2
9
  2  p 2  r 2  4 Rr   2  4 R  r   9r 2  4 R  r   3 p 2  8 R  3r  , care
3

r 4
rezultă din inegalitatea lui Gerretsen: p 2  4R 2  4 Rr  3r 2 .
Rămâne să arătăm că:
3
2  4 R  r   9r 2  4 R  r   3  4R 2  4 Rr  3r 2  8R  3r   8 R 3  9 R 2 r  12 Rr 2  4r 3  0 
  R  2r   8 R 2  7 Rr  2 r 2   0 , evident din inegalitatea lui Euler R  2 r .
Egalitatea are loc dacă și numai dacă triunghiul este echilateral.
Remarcă.
Inegalitatea poate fi întărită:
3) In  ABC
ra2 rb2 rc2 27 a 3  b 3  c3
    .
B C C A A B 4 abc
tg tg tg tg tg tg
2 2 2 2 2 2
Marin Chirciu
Soluție:
Folosind Lema și a 3  b 3  c 3  2 p  p 2  3r 2  6 Rr  inegalitatea se scrie:

27 2 p  p  3r  6 Rr 
3 2 2
 4R  r   12Rp 2
 
3
 4  4 R  r   27 r 2  6 R  3r   3 p 2 16 R  9r 
r 4 2p
care rezultă din inegalitatea lui Gerretsen: p 2  4 R 2  4 Rr  3r 2 .
Rămâne să arătăm că:
3
4  4 R  r   27r 2  6 R  3r   3  4 R 2  4 Rr  3r 2  16 R  9r  
 32 R 3  54 R 2 r  21Rr 2  2r 3  0   R  2r   32 R 2  10 Rr  r 2   0 , evident din
inegalitatea lui Euler R  2r . Egalitatea are loc dacă și numai dacă triunghiul este echilateral.
Remarcă.
Inegalitatea 3) este mai tare decât inegalitatea 1):
4) In  ABC
ra2 rb2 rc2 27 a 3  b3  c3 9 2 2 2
     a  b  c  .
B C C A A B 4 a bc 4
tg tg tg tg tg tg
2 2 2 2 2 2
Soluție:
27 a3  b3  c3 9 2 2 2
Se folosește inegalitatea 3) și   a  b  c  
4 abc 4
3 a  b  c    a  b  c   a  b  c  
3 3 3 2 2 2

29 ROMANIAN MATHEMATICAL MAGAZINE NR. 22


Romanian Mathematical Society-Mehedinți Branch 2019
2
 2 a 3   ab  a  b  , care rezultă din a3  b3  ab  a  b    a  b  a  b   0 și
analoagele.
Egalitatea are loc dacă și numai dacă triunghiul este echilateral.
Remarcă.
Să obținem o inegalitate de sens contrar:
5) In  ABC
ra2 rb2 rc2 81R
B C

C A

A B

8r
 9 R 2  32r 2  .
tg tg tg tg tg tg
2 2 2 2 2 2
Marin Chirciu
Soluție:
Folosind Lema inegalitatea se scrie:
3
 4R  r   12Rp 2
81R R
r 8r
 9 R 2  32r 2  , care rezultă din inegalitatea lui Euler r  și
2
inegalitatea lui Mitrinovic p 2  27 r 2 .
Egalitatea are loc dacă și numai dacă triunghiul este echilateral.
Remarcă.
Se poate scrie dubla inegalitate:
27 ra2 rb2 rc2 81R
6) In  ABC :
2
 5 Rr  4r 2  
B C

C A

A B

8r
 9 R 2  32r 2  .
tg tg tg tg tg tg
2 2 2 2 2 2
Marin Chirciu
Soluție:
Se folosesc 3) , 5) și inegalitatea lui Gerretsen p 2  16 Rr  5r 2 .Egalitatea are loc dacă și
numai dacă triunghiul este echilateral.
Remarcă: În același registru se pot propune:
7) In  ABC
ra2 rb2 rc2
   r  4R  r  .
B C C A A B
ctg ctg ctg ctg ctg ctg
2 2 2 2 2 2
Soluție:
S B C p
Folosind ra  și ctg ctg  obținem:
pa 2 2 pa
S2
2
ra2  p  a  S 2 1 S 2 4R  r
 B C  p 
p
 p  a p  S  r  4R  r  .

ctg ctg
2 2 pa
8) In  ABC :
ra2 rb2 rc2 9 Rr
9r 2     .
B C C A A B 2
ctg ctg ctg ctg ctg ctg
2 2 2 2 2 2
Marin Chirciu

30 ROMANIAN MATHEMATICAL MAGAZINE NR. 22


Romanian Mathematical Society-Mehedinți Branch 2019
Soluție:
ra2
Se folosește identitatea  r  4 R  r  și inegalitatea lui Euler R  2r .
B C
ctg ctg
2 2
Egalitatea are loc dacă și numai dacă triunghiul este echilateral.
9) In  ABC
ra2 r2 r2
 b  c  3 p2 .
A B C
tg 2 tg 2 tg 2
2 2 2
Soluție:
S A  p  b  p  c 
Folosind ra  și tg 2  obținem:
pa 2 p p  a
S2
2
ra2  p  a S 2 p 
1 3
 r 2 p3  2  3 p 2 .
 A

 p  b  p  c   p  a  p  b  p  c  r p
tg 2
2 p p  a
10) In  ABC
ra2 rb2 rc2 81R 2
81r 2     .
A B 2 C 4
tg 2 tg 2 tg
2 2 2
Marin Chirciu
ra2
Soluție: Se folosește identitatea   3 p 2 și inegalitatea lui Mitrinovic
A
tg 2
2
2 27 R 2
2
27 r  p  .
4
Egalitatea are loc dacă și numai dacă triunghiul este echilateral.
11) In  ABC
4
ra2 rb2 rc2 2 p 4  16 p 2 R  4 R  r    4 R  r 
   .
A B C p2
ctg 2 ctg 2 ctg 2
2 2 2
S A p p  a
Soluție: Folosind ra  și ctg 2  obținem:
pa 2  p  b  p  c 

S2
2 4
ra2  p  a S2  p  b  p  c   r 2 p 2  2 p 4  16 p 2 R  4 R  r    4 R  r 
 2A 
p p  a

p
  p  a 3 p r 2 p3

ctg
2  p  b  p  c 
4 2 4
2 p  16 p R  4 R  r    4 R  r 
 .
p2

31 ROMANIAN MATHEMATICAL MAGAZINE NR. 22


Romanian Mathematical Society-Mehedinți Branch 2019
12) In  ABC
ra2 rb2 rc2 9 Rr
   .
A B C 2
ctg 2 ctg 2 ctg 2
2 2 2
Marin Chirciu
Soluție:
4
ra2 rb2 rc22 p 4  16 p 2 R  4 R  r    4 R  r 
Folosind identitatea   
A B C p2
ctg 2 ctg 2 ctg 2
2 2 2
inegalitatea se scrie:
4
2 p 4  16 p 2 R  4 R  r    4 R  r  9 Rr 4

p2

2

 2  4 R  r   p 2 128R 2  41Rr  4 p 2 , care 
2
R  4R  r  2 2
rezultă din inegalitatea Blundon-Gerretsen 16 Rr  5r  p  .
2  2R  r 
Rămâne să arătăm că:
2
4 R  4R  r 
2  4R  r   128 R 2  41Rr  4 16 Rr  5r 2    23R 2  44 Rr  4r 2  0 
2 2R  r   
  R  2r  23R  2r   0 , evident din inegalitatea lui Euler R  2r .
Egalitatea are loc dacă și numai dacă triunghiul este echilateral.

ABOUT PROBLEM 3675-CRUX MATHEMATICORUM

By Marin Chirciu-Romania

1) Let a , b , and c be the sides of a triangle and let s be its semiperimeter. Let r and R
denote its iradius and circumradius respectively. Prove that
b  s  b   c  s  c  3R
6   .
cyclic as  a r
Proposed by Michel Bataille, Rouen, France
Soluție:
În articol voi prezenta o soluție diferita de soluția autorului apărută în Crux Mathematicorum
Vol. 38 No.8 din 2012 și voi întări dubla inegalitate din enunț.Voi demonstra rezultatul
ajutător:
Lema:
3
b  s  b   c  s  c   4 R  r   s 2  r  2R 
2) In  ABC :  a  s  a 
cyclic 2 Rs 2
.

Demonstrație:
b  s  b  c  s  c  bc  s  b  s  c  b  s  b   c  s  c  

cyclic a s  a

abc  s  a  s  b  s  c 

32 ROMANIAN MATHEMATICAL MAGAZINE NR. 22


Romanian Mathematical Society-Mehedinți Branch 2019
3
2 r 3  4 R  r   s 2  r  2 R    4 R  r  3  s 2  r  2 R 
   .
2
4 Rrs  r s 2 Rs 2
Să trecem la rezolvarea problemei propuse.
Inegalitatea din stânga, ținând seama de Lemă se scrie:
3
 4R  r 
 s2  r  2R  3
 6   4 R  r   p 2 14 R  r  ,care rezultă din inegalitatea Blundon-
2
2Rs
2
2 R  4R  r 
Gerretsen: p  . Rămâne să arătăm că:
2  2R  r 
2
3 R  4R  r 
 4R  r   14R  r   2 R 2  3Rr  2r 2  0   R  2r  2 R  r   0 , evident
2  2R  r 
din inegalitatea lui Euler: R  2r .Egalitatea are loc dacă și numai dacă triunghiul este
echilateral. Inegalitatea din dreapta, ținând seama de Lemă se scrie:
3
 4R  r 
 s 2  r  2 R  3R

3
 p 2  6 R 2  2 Rr  r 2   r  4R  r  ,
2
2 Rs r
care rezultă din inegalitatea Gerretsen: p 2  16 Rr  5r 2 .Rămâne să arătăm că:
3
16 Rr  5r  6R  2Rr  r   r  4R  r   16 R  23R r  19 Rr  2r  0
2 2 2 3 2 2 3

  R  2r  16 R  9 Rr  r   0 , evident din inegalitatea lui Euler: R  2r .
2 2

Egalitatea are loc dacă și numai dacă triunghiul este echilateral.


Remarcă: Inegalitatea din stânga poate fi întărită:
bs  b  c s  c 4  r
3) In  ABC :   5   .
cyclic as  a 3 R
Soluție: Folosind Lema inegalitatea se scrie:
3
 4R  r   s2  r  2R  4 r 3 2
  5    3  4 R  r   p  46 R  11r  ,
2 Rs 2 3 R
2
R  4R  r  2
care rezultă din inegalitatea Blundon-Gerretsen: p  .
2  2R  r 
Rămâne să arătăm că:
2
R  4R  r 
3
3 4 R  r    46R  11r   2 R 2  Rr  6r 2  0   R  2r  2R  3r   0 ,
2  2R  r 
evident din inegalitatea lui Euler: R  2r .
Egalitatea are loc dacă și numai dacă triunghiul este echilateral.
Remarcă: Inegalitatea 3) este mai tare decât inegalitatea stângă din enunț:
b s  b  c s  c 4  r
4) In  ABC :   5    6 .
cyclic as  a 3 R
4 r
Soluție: Vezi inegalitatea 3) și  5    6  R  2r ( inegalitatea lui Euler).Egalitatea are
3 R
loc dacă și numai dacă triunghiul este echilateral.
Remarcă: Inegalitatea 3) poate fi și ea întărită:

33 ROMANIAN MATHEMATICAL MAGAZINE NR. 22


Romanian Mathematical Society-Mehedinți Branch 2019
2
b s  b  c s  c 3 r r
5) In  ABC : 
cyclic a  s  a 
 7     .
2 R R
Soluție:
Folosind Lema inegalitatea se scrie:
3 2
 4R  r   s2 r  2R  3 r r 3

2 Rs 2
 
 7       R  4R  r   2 p 2 8R 2  2 Rr  r 2  ,
2 R R
2
2 R  4R  r 
 p  (inegalitatea Blundon-Gerretsen).Egalitatea are loc dacă și numai dacă
2  2R  r 
triunghiul este echilateral.
Remarcă.
Inegalitatea 5) este mai tare decât inegalitatea 3):
2
bs  b  c s  c 3 r  r  4 r
6) In  ABC :   7      5   6 .
cyclic a s  a 2 R  R 3 R
2
3 r  r  4 r
Soluție: Vezi inegalitatea 5) , inegalitatea 4) și 7        5   ,
2 R  R  3 R
adevărată din inegalitatea lui Euler: R  2r .Egalitatea are loc dacă și numai dacă triunghiul
este echilateral.
Remarcă.
Să întărim și inegalitatea din dreapta:
b  s  b   c  s  c  3 r 2R
7) In  ABC :     .
cyclic as  a 2 R r
Soluție:
Folosind Lema inegalitatea se scrie:
3
 4R  r 
 s2  r  2R  3 r 2R
  
3
 p 2  4 R 2  5Rr  r 2   r  4 R  r  ,
2 Rs 2 2 R r
care rezultă din inegalitatea Gerretsen: p 2  16 Rr  5r 2 .Rămâne să arătăm că:
3
16 Rr  5r  4R2 2
 5Rr  r 2   r  4R  r   4 R 2  7 Rr  2 r 2  0 
  R  2r  4 R  r   0 , evident din inegalitatea lui Euler: R  2r .
Egalitatea are loc dacă și numai dacă triunghiul este echilateral.
Remarcă.
Inegalitatea 7) este mai tare decât inegalitatea inegalitatea dreaptă din enunț:
b  s  b   c  s  c  3 r 2 R 3R
8) In  ABC :      .
cyclic a s  a 2 R r r
3 r 2 R 3R
Soluție: Vezi inegalitatea 7) și    , adevărată din inegalitatea lui Euler: R  2 r .
2 R r r
Egalitatea are loc dacă și numai dacă triunghiul este echilateral.
Remarcă: Se poate scrie dubla inegalitate care întărește rezultatul din enunț:
2
3 r r b  s  b   c  s  c  3 r 2R
9) In  ABC : 7           .
2 R  R  cyclic as  a 2 R r

34 ROMANIAN MATHEMATICAL MAGAZINE NR. 22


Romanian Mathematical Society-Mehedinți Branch 2019
Marin Chirciu
Soluție: Vezi inegalitatea 5) și inegalitatea 7).Egalitatea are loc dacă și numai dacă triunghiul
este echilateral.
Remarcă: Se poate scrie șirul de inegalități:
10) In  ABC :
2
4 r 3 r r b  s  b   c  s  c  3 r 2 R 3R
6  5    7           .
3 R 2 R  R  cyclic a s  a 2 R r r
Soluție: Vezi inegalitatea 6) și inegalitatea 8). Egalitatea are loc dacă și numai dacă triunghiul
este echilateral.

ABOUT PROBLEM JP.173


ROMANIAN MATHEMATICAL MAGAZINE-SPRING EDITION 2019

By Marin Chirciu-Romania
1) Let ABC be a triangle. Prove that
1 1 1 6R
   .
sin A sin B sin C r
Proposed by Nguyen Viet Hung-Hanoi-Vietnam
Soluție: Demonstrăm rezultatul ajutător:
Lemă.
2) In  ABC :
1 1 1 p 2  r 2  4 Rr
   .
sin A sin B sin C 2rp
Demonstrație:
a
Folosind sin A  ,  bc  p 2  r 2  4 Rr și abc  4 Rrp , obținem:
2R
1
 sin A  a

2R
 2 R a
1
 2 R   bc
 2 R 
p 2  r 2  4 Rr p 2  r 2  4 Rr
 .
abc 4 Rrp 2rp
Să trecem la rezolvarea problemei din enunț:
Folosind Lema inegalitatea de demonstrat se scrie:
2
p 2  r 2  4 Rr 6R  p 2  r 2  4 Rr  6 R
     
2rp r  2rp  r
2
p 4  2 p 2  r 2  4 Rr   r 2  4 R  r  6R
  
4r 2 p 2 r

35 ROMANIAN MATHEMATICAL MAGAZINE NR. 22


Romanian Mathematical Society-Mehedinți Branch 2019
2
 p 2  p 2  2r 2  16 Rr   r 2  4 R  r   0 .
Distingem cazurile:
Cazul 1). Dacă  p 2  2r 2  16 Rr   0 , inegalitatea este evidentă.
Cazul 2). Dacă  p 2  2r 2  16 Rr   0 , inegalitatea se rescrie:
2
r 2  4 R  r   p 2 16Rr  2r 2  p 2  , care rezultă din inegalitatea Blundon-Gerretsen:
2
2 R  4R  r 
2
16Rr  5r  p  . Rămâne să arătăm că :
2  2R  r 
2
R  4R  r  3 Rr 2
2
r 2  4R  r  
2  2R  r 

16 Rr  2r 2  16 Rr  5r 2   r 2  
2  2R  r 
 2  2 R  r   3R

 R  2 r , (inegalitatea lui Euler).


Egalitatea are loc dacă și numai dacă triunghiul este echilateral.
Remarcă:Să punem în evidență o inegalitate de sens contrar:
3) In  ABC :
1 1 1 R 3
   .
sin A sin B sin C r
Marin Chirciu
Soluție: Folosind Lema inegalitatea se scrie:
2
p 2  r 2  4 Rr R 3  p 2  r 2  4 Rr  2
    3R 
2rp r  2 p 
2
p 4  2 p 2  r 2  4 Rr   r 2  4 R  r  2
 2
 3R 2  r 2  4 R  r   p 2 12 R 2  8Rr  2r 2  p 2  ,
4p
care rezultă din inegalitatea Gerretsen:
2
r  4R  r 
 16Rr  5r 2  p 2  4 R 2  4 Rr  3r 2 . Rămâne să arătăm că :
Rr
2
r  4R  r 
2
r 2  4R  r  
Rr

12 R 2  8Rr  2r 2   4R 2  4 Rr  3r 2   
r  R  r   8R 2  12 Rr  5r 2  8 R 2  13 Rr  6 r 2  0   R  2r  8R  3r   0 , evidentă
din inegalitatea lui Euler R  2r .Egalitatea are loc dacă și numai dacă triunghiul este
echilateral.
Remarcă: Se poate scrie dubla inegalitate:
4) In  ABC :
6R 1 1 1 R 3
    .
r sin A sin B sin C r

Soluție: Vezi inegalitățile 2) și 3). Egalitatea are loc dacă și numai dacă triunghiul este
echilateral.

36 ROMANIAN MATHEMATICAL MAGAZINE NR. 22


Romanian Mathematical Society-Mehedinți Branch 2019

ABOUT INEQUALITY IN TRIANGLE-884


ROMANIAN MATHEMATICAL MAGAZINE
By Marin Chirciu-Romania

1) In  ABC
ma mb mc 3
   .
bc ca ab 2 R
Daniel Sitaru
Soluție.
ma ma 1 ma 1 3
Folosind bc  2 Rha și ma  ha obținem  bc   2Rh 
2R
 
ha 2 R
3 
2R
.
a
Egalitatea are loc dacă și numai dacă triunghiul este echilateral.
Remarcă: Să punem în evidență o inegalitate de sens contrar.
2) In  ABC

ma mb mc 3
   .
bc ca ab 4 r
Marin Chirciu
Soluție.
Folosind bc  2 Rha obținem
m m 1 m
 bca   2 Rha  2 R  h a , (1).
a a

1 1 1 
Tripletele  ma , mb , mc  și  , , ,  sunt invers ordonate. Cu inegalitatea lui Cebâșev
 ha hb hc 
obținem:
m 1 1 1 9 R 1 3R
 h a  3  ma  h  3  2  r  2r , ( 2).
a a

Din (1) și (2) rezultă concluzia.


Egalitatea are loc dacă și numai dacă triunghiul este echilateral.
Remarcă.
Se poate scrie dubla inegalitate:
3) In  ABC
3 m m m 3
 a b c .
2 R bc ca ab 4 r
Daniel Sitaru,Marin Chirciu

37 ROMANIAN MATHEMATICAL MAGAZINE NR. 22


Romanian Mathematical Society-Mehedinți Branch 2019
Soluție:Vezi inegalitățile 1) și 2).Egalitatea are loc dacă și numai dacă triunghiul este
echilateral.
Remarcă:Dubla inegalitate poate fi întărită:
4) In  ABC

1  7 r  ma mb mc 1  r 
      2  .
2 R  2 R  bc ca ab 3r  2R 
m m m 1 7 r 
Soluție: Inegalitatea din stânga : a  b  c     rezultă din:
bc ca ab 2 R  2 R 
m m 1 m
 bca   2 Rha  2 R  h a , (1).
a a

b2  c 2 bc b 2  c 2 p 2  r 2  2 Rr
Cu inegalitatea lui Tereshin ma  , ha  ,   și
4R 2R bc 2 Rr
inegalitatea lui Gerretsen: p 2  16 Rr  5r 2 obținem:
b2  c 2
2 2 2 2 2 2 2 2
ma 4 R  1 b  c  1  p  r  2 Rr  p  r  2 Rr  p  r  2 Rr 
 h  bc 2  bc 2

2 Rr 4 Rr 4 Rr
a
2R
16 Rr  5r  r 2  2 Rr 14 Rr  4r 2 7 R  2r 7 r
2
     , (2).
4Rr 4 Rr 2R 2 R
ma mb mc 1 7 r 
Din (1) și (2) obținem      .
bc ca ab 2 R  2 R 
m m m 1 r 
Inegalitatea din dreapta: a  b  c   2   . Folosind bc  2Rha obținem
bc ca ab 3r  2R 
m m 1 m 1 1 1 
 bca   2 Rha  2 R  h a , (1). Tripletele  ma , mb , mc  și  h , h , h ,  sunt invers
a a  a b c 
ordonate. Cu inegalitatea lui Cebâșev obținem:
m 1 1 1 1 4R  r
 h a  3  ma  h  3   4 R  r   r  3r , ( 2).
a a

ma mb mc 1 4R  r 4R  r 1  r 
Din (1) și (2) rezultă       2 .
bc ca ab 2 R 3r 6 Rr 3r  2R 
Egalitatea are loc dacă și numai dacă triunghiul este echilateral.
Remarcă: Dubla inegalitate 4) este mai tare decât 3).
5) In  ABC

3 1  7 r  ma mb mc 1  r  3
       2  .
2 R 2 R  2 R  bc ca ab 3r  2 R  4r
Soluție: Vezi 4) și inegalitatea lui Euler R  2r .Egalitatea are loc dacă și numai dacă triunghiul
este echilateral.

38 ROMANIAN MATHEMATICAL MAGAZINE NR. 22


Romanian Mathematical Society-Mehedinți Branch 2019

POLYNOMES AND MATRICES


By Marian Ursărescu-Romania
Within this article we’ll try to solve problems which ask to demonstrate that the determinant
of a matrice is positive.First let’s remind the following properties of the numbers:
1. + + ⋯ + = + + ⋯ + , ∀ , , … , ∈ ℂ∗
2. ⋅ ⋅ … ⋅ = ⋅ ⋅ … ⋅ , ∀ , , … , ∈ ℂ∗
3. | | = ⋅ , ∀ ∈ ℂ∗
Theorem 1 : 1. det = det , ∀ ∈ ℳ (ℂ) 2. det ⋅ ≥ 0, ∀ ∈ ℳ (ℂ)
Demonstration:
1. Consider = =

det = ( ) ( ) ( )… ( )

det = ( ) ( ) ( ) … ( ) =

= ( ) ( ) ( ) … ( ) = ( ) ( ) ( )… ( ) = det
∈ ∈
2. det ⋅ = det ⋅ det = det ⋅ det = |det | ≥ 0

Theorem 2: Consider ∈ ℝ[ ], ( ) = + + , write > 0 and Δ = − 4 < 0.


Then det( + + ) ≥ 0, ∀ ∈ ℳ (ℝ).
Demonstration: If Δ < 0 then has conjugated complex roots: , ∈ ℂ, with =
det[( − )( − )] = det[( − )( − )] = det[( − )( − )] ≥ 0
(according to theorem 1)
On the other hand: det[( − )( − )] = det[ − ( + ) + ]=
1
= det + + = det( + + )
Consequently det( + + ) ≥ 0.The generalization of theorem 2
Consider ∈ ℝ[ ], ( ) = + + ⋯+ + , > 0 and all its roots
are complexe. Then + ⋯+ + ≥ 0, ∀ ∈ ℳ (ℝ)
Demonstration: If has all roots complex (numbers) and ∈ ℝ[ ] then
( )= ( − )( − ) … − − and we can use the second theorem for
the polynome ( − )( − ), = 1,

39 ROMANIAN MATHEMATICAL MAGAZINE NR. 22


Romanian Mathematical Society-Mehedinți Branch 2019
Theorem 3: Consider ∈ ℝ[ ], ( ) = + + ,Δ = − 4 < 0, ∀ , ∈ ℳ (ℝ) the
following statement is true: det[( + )( + )] ≥ 0, , being the roots of the
polynome .
Demonstration: If Δ < 0 then , ∈ ℂ and =
det[( + )( + )] = det[( + )( + )] = det[( + )( + )] ≥ 0
(according to theorem 1)
Notice that if = then the conclusion of the third theorem can be written like that:
det( − + )≥0
Demonstration: det[( + )( + )] = det[ + ( + ) + ]=
= det ( − + ) ≥ 0, (we used = and Viète’s relations)

Theorem 4: Consider ∈ ℝ[ ], ( ) = + + ,Δ = − 4 < 0, ∀ , ∈ ℳ (ℝ) the


following statement is true: det[( + + )( + + )] ≥ 0, , being the
roots of .
Demonstration: If Δ < 0 then , ∈ ℂ and =
det[( + + )( + + )] = det[( + + )( + + )] =
= det [( + + )( + + )] ≥ 0 (according to the first theorem)
If in addition, = then the conclusion of the fourth theorem can be written like that:
det[ + ( + ) − ( + ) + ( − 2 ) ] ≥ 0
Demonstration: det[( + + )( + + )] =
= det[ + ( + ) + ( + )( + ) + ( + ) ] =
= det[ + ( + ) − ( + ) + ( + 2 ) ] (we used = and Viète’s
relations)

Theorem 5: Consider ∈ ℝ[ ], ( ) = + + ,Δ = − 4 < 0, ∀ , ∈ ℳ (ℝ) the


following statement is true: det[( + + )( + + )] ≥ 0, , being the
roots of .
Demonstration: If Δ < 0 then , ∈ ℂ and =
det[( + + )( + + )] = det[( + + )( + + )] =
= det[( + + )( + + )] ≥ 0 (according to the first theorem)
If = , = , = , then the conclusion of this theorem can be written this
way: det[ + ( + ) − ( + )+( +2 ) ] ≥ 0
Demonstration: det[( + + )( + + )] =
= det[ + ( + ) + ( + )( + )+( + ) ]=
= det[ + ( + ) − ( + )+( +2 ) ]≥ 0
(we used = , = , = and Viète’s relations). Now let’s solve some
problems related to these theorems.
1. Show that det( + ) ≥ 0, ∀ ∈ ℳ (ℝ)
Demonstration: We use theorem 2 for ( ) = + 1, whose roots are = , = −

2. Show that det( + + ) ≥ 0, ∀ ∈ ℳ (ℝ)


Demonstration: We use theorem 2 for ( ) = + + 1, with the roots
−1 + √3 −1 − √3
= , =
2 2

3. If ∀ , ∈ ℳ (ℝ) so that = , then det( + )≥0


40 ROMANIAN MATHEMATICAL MAGAZINE NR. 22
Romanian Mathematical Society-Mehedinți Branch 2019
Demonstration: We use the third theorem for ( ) = + 1, where + = 0, =1

4. If , ∈ ℳ (ℝ) so that = , then det( + 2 +2 )≥0


Demonstration: Using the third theorem we consider
( )= + 2 + 2, + = −2, =2

5. If , ∈ ℳ (ℝ), + = , then det( + ) ≥ 0


Demonstration: Let’s show first that = , + = , we multiply right by
⇒ + = (1)
+ = , we multiply left by ⇒ + = (2)
+ = ℎ = −
⇒ =
+ = = −
det( + ) = det[( + )( − + )] = det( + ) det( − + )=
= det( − + ) ≥ 0 (using theorem 3 for ( ) = − + 1)

6. If , , ∈ ℳ (ℝ), = , = , = then
det( + + − − − )≥0
Demonstration: We use theorem 5 for ( ) = + + 1, whose roots are , ∈ ℂ with
+ = −1, = 1 and + = −1.
7. If , ∈ ℳ (ℝ), = then det(2 + 2 − 3 − − + )≥0
Demonstration: We use theorem 4 for ( ) = − + 2, with the roots , ∈ ℂ, +
= 1, = 2 and = 2 and + = −3.

8. If , , ∈ ℳ (ℝ), = , = , = , then
det( + ( + )( + ) + 2 ( + ) + 2( + ) ) ≥ 0, ∀ , ∈ ℝ
Demonstration: Using theorem 5 we consider ( ) = + 2 + + , Δ = −4 ≤ 0,
and the roots , ∈ ℂ , + = 2 , = + , + = 2( + ).
9. If , ∈ ℳ (ℝ), = , then:
det( + 2( + )( + ) + 2( + )( + ) + 8 ) ≥ 0, ∀ , ∈ ℝ
Demonstration: We use theorem 4 for, ( ) = − 2( + ) + 2 + 2 ,
Δ = −4( − ) ≤ 0, , ∈ ℂ, with + = 2( + ) , + = 2( + )
10. If ∈ ℳ (ℝ), then det( + + + ⋯+ ) ≥ 0, ∀ ∈ ℕ∗
Demonstration: We use the generalization of theorem 2 for ( ) = 1 + + + ⋯ +
which has only complex roots
BIBLIOGRAPHY: Maths Review

PROPOSED PROBLEMS
5-CLASS-STANDARD

41 ROMANIAN MATHEMATICAL MAGAZINE NR. 22


Romanian Mathematical Society-Mehedinți Branch 2019

V.1. Prove that 1x abc if and only if 1x x  ab  c .Proposed by Marin Chirciu – Romania

V.2. Prove that 7 4 n 3  37 n5 , where n  N . Proposed by Marin Chirciu – Romania

V.3. Let be the number A   2n  11  2  3  ...  2n  , n  N . Prove that


2
A  n  2n  1 , n  N . Prove that if n  5k 2  10, k  N , then A can be written as a sum of
five perfect squares, nonzero and distinct, for k ≥ 3.

Proposed by Marin Chirciu – Romania

V.4. Find all natural numbers having maximum six digits that have the digits directly
proportional with their rank from the right to the left.

Proposed by Daniel Sitaru-Romania

V.5. Prove that it exists an infinity of natural numbers  x, y, z  for which the number

13 x  13 y  13z to be a perfect cube. Proposed by Marin Chirciu – Romania

V.6. Find the natural numbers n such that the sum 1  2  ...  n to be a number formed
from two identical digits. Proposed by Titu Zvonaru, Neculai Stanciu-Romania

V.7. Let be the natural numbers a, b, c , x, y, z which verifies the relationships a  xy  x  y


, b  xz  x  z , c  yz  y  z . Prove that the product of the successors of the numbers
a, b, c is a perfect square.
Proposed by D.M. Bătineţu-Giurgiu,Neculai Stanciu-Romania
V.8. Find all the numbers a879b divisible with 72.
Proposed by D.M. Bătineţu-Giurgiu, Neculai Stanciu-Romania

V.9. Find all the numbers abcde for which abcde  4  edcba .
Proposed by D.M. Bătineţu-Giurgiu, Neculai Stanciu-Romania

V.10. Prove that the nonzero natural numbers a, b, c satisfy the relationship
43a 2  129b  25c  1720, then they are prime.

Proposed by Titu Zvonaru, Neculai Stanciu-Romania


V.11. Prove that the number 194800…007171, which has in total 2016 zeros, is composed.
Proposed by D.M. Bătineţu-Giurgiu, Neculai Stanciu-Romania

V.12. If to different letters correspond different digits, then find all the numbers abc
knowing that the arithmetic means of the numbers abc , acb , bac , bca , cab and cba is abc .

42 ROMANIAN MATHEMATICAL MAGAZINE NR. 22


Romanian Mathematical Society-Mehedinți Branch 2019

Proposed by D.M. Bătineţu-Giurgiu, Neculai Stanciu-Romania

V.13. Solve the equation x ( x  1)( x  2)  abcabc .


Proposed by D.M. Bătineţu-Giurgiu, Neculai Stanciu-Romania

V.14. If to different letters correspond different digits then find all the numbers abc which
verify the relationship abc  ab  ba  bc  cb  ca  ac .
Proposed by D.M. Bătineţu-Giurgiu, Neculai Stanciu-Romania

V.15. If to different letters correspond different digits such that


adcb  bcda  cabd  dbac  abbba ,then find abcd .

Proposed by D.M. Bătineţu-Giurgiu, Neculai Stanciu-Romania


V.16. If to different letters corresponds different digits and cd  2  ab , then find all the
perfect squares having the form abcdab .
Proposed by D.M. Bătineţu-Giurgiu, Neculai Stanciu-Romania
V.17. In how many ways can we weight 10 grams with a balance if we have 8 weights of one
gram, 3 of two grams and 4 of three grams?
Proposed by D.M. Bătineţu-Giurgiu, Neculai Stanciu-Romania
V.18. What age is Bogdan, knowing that his age is one year bigger than eight times the sum
of the digits that it is formed?
Proposed by D.M. Bătineţu-Giurgiu, Neculai Stanciu-Romania
V.19. Prove that if p is odd number and not divisible with 3 then the number p 2  2015
divides with 24.
Proposed by D.M. Bătineţu-Giurgiu, Neculai Stanciu-Romania
V.20. How many digit has the number 2 96 ?

Proposed by Titu Zvonaru, Neculai Stanciu-Romania


V.21. Suma a 12 numere naturale consecutive este un număr cu cifre diferite, printre care se
află cifrele 1,2,3 și 4. Care este cel mai mic număr posibil dintre cele 12 numere?
Proposed by Ionică Constantin-Romania

V.22. Determinați numărul , știind că 4 + 1026 =


Proposed by Ionică Constantin-Romania

V.23. Scrieți numărul 164 ca sumă de patru pătrate perfecte.


Proposed by Ionică Constantin-Romania

V.24. Se consideră numărul = 7+ 7 +7 +⋯+ 7 . Arătați că 6 + 7 este pătrat


perfect. Proposed by Ionică Constantin-Romania

V.25. Câte numere naturale de patru cifre se pot scrie ca sumă a cinci numere naturale
consecutive? Proposed by Ionică Constantin-Romania

43 ROMANIAN MATHEMATICAL MAGAZINE NR. 22


Romanian Mathematical Society-Mehedinți Branch 2019

V.26. Să se arate că dacă 23 ⋅ = + 369, atunci numărul se divide cu 41.


Proposed by Ionică Constantin-Romania
All solutions for proposed problems can be finded on the
http//:www.ssmrmh.ro which is the adress of Romanian Mathematical
Magazine-Interactive Journal.
6-CLASS-STANDARD

VI.1. Find all the pairs ( x, y) of integers which verify the relationship x 2  xy  y  2018 .
Proposed by D.M. Bătineţu-Giurgiu,Neculai Stanciu-Romania

VI.2. Let be a, b, c, d , e, f real numbers, strictly positive which verify the relationships
(i) a 2  b 2  c 2  16 ; (ii) d 2  e 2  f 2  36 ; (iii) ad  be  cf  24 .
abc 2
Find  . Proposed by D.M. Bătineţu-Giurgiu,Neculai Stanciu-Romania
d e f 3

VI.3. If p  5 is a prime number, then find the reminder of the number division p 2 to 12.
Proposed by D.M. Bătineţu-Giurgiu,Neculai Stanciu-Romania

VI.4. If a and b are digits, then solve the equation ( ab  ba )( ab  ba )  x 2 .

Proposed by D.M. Bătineţu-Giurgiu,Neculai Stanciu-Romania

x 2  xy  y 2 7
VI.5. Find all the pairs ( x, y )  Z  Q , which verify the relationship  .
x  2y 5

Proposed by D.M. Bătineţu-Giurgiu,Neculai Stanciu-Romania

VI.6. Find all the integers a for which a 2  8a is a perfect square.

Proposed by D.M. Bătineţu-Giurgiu,Neculai Stanciu-Romania


 18  9 x 8 
VI.7. Find the set A   x  Z 4
 Z .
 4  3x 

Proposed by Neculai Stanciu,Iuliana Traşcă-Romania

44 ROMANIAN MATHEMATICAL MAGAZINE NR. 22


Romanian Mathematical Society-Mehedinți Branch 2019
VI.8. Let be the isosceles triangle ABC with AB  AC , the point D on the line BC , with C
between B and D and the point E the symmetric of the point B towards the line AD .
Prove that BAD  ECD . Proposed by Titu Zvonaru,Neculai Stanciu-Romania

VI.9. Prove that it doesn’t exists a prime number p such that both numbers p 3  2015 and
p 3  2017 to be prime. Proposed by Neculai Stanciu,Titu Zvonaru-Romania

VI.10. Fie A= 472018 , arătați că :

a) A nu se poate scrie ca sumă a două cuburi de numere întregi


b) A nu se poate scrie ca suma a trei cuburi de numere întregi.
Proposed by Cremeneanu Luiza Lorena, Prunaru Constantina-Romania

VI.11. If a, b  Z such that 25a  12b is a multiple of 11, then 2a  3b is a multiple of 11.

Proposed by Marin Chirciu-Romania


 1  1  1 
VI.12. Solve in nonzero natural numbers set the following equation 1  1  1    2
 a  b  c 
Proposed by D.M. Bătineţu-Giurgiu,Neculai Stanciu-Romania

VI.13. If to different letters correspond different digits, then find all the digits a, b, c which
verify the relationship ab  ba  cbac .
Proposed by D.M. Bătineţu-Giurgiu,Neculai Stanciu-Romania
VI.14. From 100 pupils of a school, that have participated at baccalaureate exam, 70 passed
test A, 80 test B, 75 test C and respectively 85 test D. Prove that at least 10 pupils have
passed this exam.
Proposed by D.M. Bătineţu-Giurgiu,Neculai Stanciu-Romania
VI.15. Prove that the number 102400…0081, which has in total 2015 zeros, is composed

Proposed by D.M. Bătineţu-Giurgiu,Neculai Stanciu-Romania


VI.16. Prove that for any natural number n 1  n  2012 , it exists a natural number m
such that all the numbers m  2 n  1 to be composed.

Proposed by D.M. Bătineţu-Giurgiu,Neculai Stanciu-Romania


VI.17. Find all the triplets  x, y, z  of natural numbers with 0  x  y  z , which verify the
 
relationship x 2  y 2  z 2  m m  N * .

Proposed by D.M. Bătineţu-Giurgiu,Neculai Stanciu-Romania


1
VI.18. If a  , then compute the whole part of the number
2011
1 1 1
S  2
 ...  .
1 a 1 a  a 1  a  a  ...  a 2011
2

Proposed by D.M. Bătineţu-Giurgiu,Neculai Stanciu-Romania

45 ROMANIAN MATHEMATICAL MAGAZINE NR. 22


Romanian Mathematical Society-Mehedinți Branch 2019

VI.19. If a, b, c  R * such that a  b  c  ab  bc  ca  3 , then find:

a 2011  b 2011  c 2011 .


Proposed by D.M. Bătineţu-Giurgiu,Neculai Stanciu-Romania
VI.20. Let a, b, c be the nonzero real numbers such that a  b  c  9 and ab  bc  ca  27 .
Find a 2014  b 2014  c 2014 .

Proposed by D.M. Bătineţu-Giurgiu,Neculai Stanciu-Romania


⋅ ⋅ ⋅…⋅
VI.21. Arătați că fracția ⋅ ⋅ ⋅…⋅
este ireductibilă.
Proposed by Ionică Constantin-Romania
VI.22. Fie numerele natural și . Arătați că dacă fracția se simplifică prin 17, atunci și
fracția se simplifică cu 17. Proposed by Ionică Constantin-Romania

VI.23. Arătați că nu există numere naturale , , pentru care + + = 18 și


+ + = 107. Proposed by Ionică Constantin-Romania

VI.24. Se consideră numerele raționale = și = . Arătați că + > 2,7(3).


Proposed by Ionică Constantin-Romania

VI.25. Fie , , ∈ ℕ care nu sunt divizibile cu 3. Arătați că suma + + este divizibilă


cu 3. Proposed by Ionică Constantin-Romania

VI.26. Numerele + și − sunt direct proporționale cu 12 și 3. Determinați numerele


și știind că ⋅ = 135. Proposed by Ionică Constantin-Romania

All solutions for proposed problems can be finded on the


http//:www.ssmrmh.ro which is the adress of Romanian Mathematical
Magazine-Interactive Journal.
7-CLASS-STANDARD

VII.1. Let , , be positive real numbers satisfying + + = 3. Prove that:


+ + +
+ + ≥3
+ + +
Proposed by Pham Quoc Sang-Vietnam

46 ROMANIAN MATHEMATICAL MAGAZINE NR. 22


Romanian Mathematical Society-Mehedinți Branch 2019
VII.2. Find all pairs (x, y) of integers satisfying the equation

x − (y + 2)x + (y − 1)x + (y + 2)x + y = 2.


Proposed by Nguyen Viet Hung-Hanoi-Vietnam

VII.3. Find the numbers , , ∈ ℕ∗ knowing that:

∈ ℕ, ∈ ℕ and ∈ℕ

Proposed by Gheorghe Alexe, George – Florin Șerban– Romania

VII.4. Let be a trapeze where ∥ ; = ; = ; = ; = ; > .


( )( )
Prove that Area [ ]<
( )
. Proposed by Daniel Sitaru – Romania

VII.5. Given , , > 0 and + + = 6, prove + + + + + ≥6

Proposed by Nguyen Phuc Tang– Vietnam

VII.6. Prove that the following inequalities hold for all positive real numbers:

a. + + ≥ ⋅

b. ( )
+ ( )
+ ( )
≥ ⋅

Proposed by Nguyen Viet Hung–Vietnam

VII.7. Prove that the following inequalities holds for all positive real numbers a, b, c:

( )
a. + + ≥
( )
b. + + ≥
Proposed by Nguyen Viet Hung-Vietnam

VII.8. Prove that for all positive real numbers a, b, c :

a(b + c ) b(c + a ) c(a + b ) 6abc


+ + ≥
2a + bc 2b + ca 2c + ab ab + bc + ca
Proposed by Hung Nguyen Viet-Vietnam
VII.9. Let x, y, z > 0 be positive real numbers. Then

1 1 1 4 3xyz(x + y + z)
+ + ≥
x + y y + z z + x (x + y)(y + z)(z + x)
Proposed by D.M. Bătinețu-Giurgiu-Romania, Martin Lukarevski-Macedonia

47 ROMANIAN MATHEMATICAL MAGAZINE NR. 22


Romanian Mathematical Society-Mehedinți Branch 2019

VII.10. Prove that for all positive real numbers a, b, c:

( )
a. + + ≥ + ( )

( )
b. + + ≥ + ( )

Proposed by Nguyen Viet Hung-Vietnam

VII.11 Prove that if ∈ (0, ∞); ∈ 1, ; ∈ ℕ; ≥ 3; = ; ⋅…⋅ = 1, then

+ +1
≥ √3
+ +

Proposed by Daniel Sitaru– Romania

VII.12. Solve for natural numbers: + + + ⋯+ = 3025

Proposed by Daniel Sitaru– Romania


VII.13. We consider the right triangle ABC with the right angle in A . Let D be the contact
point of altitude from A on BC and E the bisector’s intersection of the angle ADC with
AE ME
AC . If M  ( AE ), N  ( DC ) and F   MN  AD such that   k  1 , find in
DC NC
BD
function of k the rapport .
AF
Proposed by D.M. Bătineţu-Giurgiu,Neculai Stanciu-Romania
VII.14. Prove that the number = 111 … 1 is composed.

Proposed by D.M. Bătineţu-Giurgiu,Neculai Stanciu-Romania

VII.15. Prove that the number 999 … 9 + 1 999 … 9 000 … 0 is composed.


Proposed by D.M. Bătineţu-Giurgiu,Neculai Stanciu-Romania


VII.16. We consider an ABC triangle with the lengths sides AB  16, BC  34, CA  30 and
the points M respectively N on the side BC such that BM  4, MN  12 .
Compute the measure of the angle MAN .

Proposed by D.M. Bătineţu-Giurgiu,Neculai Stanciu-Romania


VII.17. We denote “Super – Heron” triangle (do not confuse with “Super – Hero”) a triangle
that has the lengths sides consecutive natural numbers and the area also a natural number.

48 ROMANIAN MATHEMATICAL MAGAZINE NR. 22


Romanian Mathematical Society-Mehedinți Branch 2019
Recent1(2007) was proved the existence of an infinite number of such triangles.
Confirm or infirm the existence of “Super – Heron” inscriptible quadrilaterals.
(Indication. Use the area formula of the incriptible quadrilateral given by the Indian
mathematician Brahamgupta in VII century - A.H: A  ( s  a)(s  b)(s  c)(s  d ) , where
a, b, c, d are the quadrilateral lengths sides, and s is its semiperimeter)
Proposed by D.M. Bătineţu-Giurgiu,Neculai Stanciu-Romania
VII.18. We consider a right-angled triangle with the perimeter P and the area A natural
numbers.Prove that: the hypotenuse is a natural number if and only if P is natural even
P
number and divides A  P 2 A .
2
Proposed by D.M. Bătineţu-Giurgiu,Neculai Stanciu-Romania
VII.19. Which conditions must meet the triangle’s angle so this can be divided in two
isosceles triangles? Find the angles of an isosceles triangle which can be divided in two
isosceles triangles. Proposed by D.M. Bătineţu-Giurgiu,Neculai Stanciu-Romania

VII.20. Find the biggest triangle with the dimensions natural numbers whose sum is even
number for which the area is also a natural number and it is equal with its perimeter.
Proposed by D.M. Bătineţu-Giurgiu,Neculai Stanciu-Romania

VII.21. Fie , , numere reale astfel încât au loc simultan relațiile: 5 − 7 + 9 ≥ 0,


7 − 9 + 11 ≥ 0, 9 − 5 − 20 ≥ 0. Arătați că 5 − 28 + 27 = 24.
Proposed by Ionică Constantin-Romania

VII.22. Să se arate că într-un triunghi cu lungimile laturilor , , și care are perimetrul egal
cu 15 este adevărată relația √ + − + √ + − + √ + − ≤ 9.
Proposed by Ionică Constantin-Romania

VII.23. Dacă , , , sunt lungimile laturilor unui patrulater și verifică relația:


= = = , stabiliți natura patrulaterului.
Proposed by Ionică Constantin-Romania

VII.24. Se știe că lungimile diagonalelor unui romb verifică relația: + = 2√2. Determinați
măsurile unghiurilor rombului. Proposed by Ionică Constantin-Romania
VII.25. Stabiliți dacă numărul:

= 1 + 2018 ⋅ 1 + 2019 ⋅ 1 + 2020 ⋅ 1 + 2021 ⋅ √1 + 2022 ⋅ 2024 este rațional

sau irațional. Proposed by Ionică Constantin-Romania

VII.26. Arătați că numărul = 2019 − 10 ⋅ 2018 − 20 ⋅ 2018 + 15 este pătrat perfect.


Proposed by Ionică Constantin-Romania

1
http://www.math.twsu.edu/~richardson/heronian/heronian.html
49 ROMANIAN MATHEMATICAL MAGAZINE NR. 22
Romanian Mathematical Society-Mehedinți Branch 2019

2 2

VII.27. Să se rezolve ecuația


a  b  2a a

a  b  2b b
 4 ab , unde a, b și c sunt strict
b a
pozitive și a  b  2 . Proposed by Carmen Chirfot-Romania

21 1 2a 2b
VII.28. Să se demonstreze că  a  b      4 ab   , unde a, b și c sunt numere reale
a b b a

strict pozitive. Proposed by Carmen Chirfot-Romania

All solutions for proposed problems can be finded on the


http//:www.ssmrmh.ro which is the adress of Romanian Mathematical
Magazine-Interactive Journal.

8-CLASS-STANDARD

VIII.1. Solve for integers:

− −
+ + = 2018201720162015
2 6
Proposed by Rovsen Pirguliyev-Sumgait-Azerbaijan

VIII.2. If = 2018 , ∈ ℕ then:
a.Prove that can not be written as a sum of two perfect cubes.
b.Prove that can not be written as a sum of three perfect cubes.
Proposed by Lucian Tutescu,Daniela Iancu– Romania

VIII.3. If , , > 0, √1 + + √1 + + √1 + = 3√2 then:

3√2
+ + ≤
√1 + √1 + √1 + 2
Proposed by Daniel Sitaru– Romania

VIII.4. If , , > 0, + + = 3 then: ∑ ( +2 )+ ( + 2 ) ≤ 6√3

Proposed by Daniel Sitaru– Romania

50 ROMANIAN MATHEMATICAL MAGAZINE NR. 22


Romanian Mathematical Society-Mehedinți Branch 2019

VIII.5. Solve in R 2 the system of equations:


2011
( x  x 2  1)( y  y 2  1)  2012 ; x  y 
.
2012
Proposed by D.M. Bătineţu-Giurgiu,Neculai Stanciu-Romania
VIII.6. If , , > 0, + + = 1 then: ∑( + ) ≥ 4 3

Proposed by Daniel Sitaru– Romania

VIII.7. Prove that if a, b, c > 0 then:

a b c a 2b 4c
+2 +4 ≤ 7 + +
b+c c+a a+b b+c c+a a+b
Proposed by Daniel Sitaru– Romania
VIII.8. If , , >, + + = 3 then:

+ −1
≥ + +
+ −1
Proposed by Daniel Sitaru – Romania

VIII.9. If , , > 0, + + = 3 then:

+ + <1+ + +
+ +1 + +1 + +1
Proposed by Daniel Sitaru-Romania

VIII.10. If , , > 0, + + = 1 then:

+ + +2 + + ≥3
2
Proposed by Daniel Sitaru-Romania
VIII.11. If , , > 0 then:
+ + +
+ + ≤ + +
+ + +
Proposed by Daniel Sitaru-Romania

VIII.12. If , , > 0, = 1 then:

( −1+ )( + )≥6
Proposed by Daniel Sitaru-Romania

VIII.13. If , , > 0 then:

+ + < 1+ + +
1+ + 1+ + 1+ +
Proposed by Daniel Sitaru – Romania
VIII.14. Solve for real numbers:
51 ROMANIAN MATHEMATICAL MAGAZINE NR. 22
Romanian Mathematical Society-Mehedinți Branch 2019
[| |] = |[ ]|
, [∗] - great integer function
[| |] = |[ ]|
Proposed by Daniel Sitaru – Romania
VIII.15. If , , > 0 and + + = 3 prove that:
1 1 18
+ ≥
+ +
where ≥ 0.
Proposed by Marin Chirciu– Romania

VIII.16. Let , , be positive real numbers. Prove that:

( − + ) ( − + ) ( − + ) 3
+ + ≥
( + ) ( + ) ( + ) 16
Proposed by George Apostolopoulos-Greece

VIII.17. Prove that if , , ∈ ℝ then:

(2 − − − + ) ≤( + 2)( + 2)( + 2)
Proposed by Daniel Sitaru – Romania
VIII.18. If , , > 0; ≥ 1 then:

3 ( + + ) + +
+ ≥ +1
( + + ) + +

Proposed by Marin Chirciu-Romania

VIII.19. If , , > 0 then:


13 13 13
+ + ≤3
6 +7 6 +7 6 +7

Proposed by Marin Chirciu– Romania

VIII.20. Let a, b, c be non-negative such that a + b + c = 3. Prove that:

3√3
|(a − b)(b − c)(c − a)| ≤
2
Equality occurs when?
Proposed by Nguyen Ngoc Tu-Vietnam

VIII.21. Rezolvați în ℝ ecuația: | − 2| ⋅ | + 5| = | + 2| ⋅ | − 5|


Proposed by Ionică Constantin-Romania
VIII.22. Fie , ∈ ℕ∗ astfel încât 61|(5 + 6 ). Arătați că 61|( + ).
Proposed by Ionică Constantin-Romania
VIII.23. Fie , , > 0 numere reale astfel încât = 3. Arătați că:
3 4 5 1
+ + ≤
(3 + )( + 3 ) (4 + )( + 4 ) (5 + )( + 5 ) 4
Proposed by Ionică Constantin-Romania

52 ROMANIAN MATHEMATICAL MAGAZINE NR. 22


Romanian Mathematical Society-Mehedinți Branch 2019

VIII.24. Rezolvați ecuația:


−1 −3 −5 −7 − 2019 − 2017 − 2015 − 2013
+ + + = + + +
2019 2017 2015 2013 1 3 5 7
Proposed by Ionică Constantin-Romania
VIII.25. Rezolvați în muțimea numerelor întregi ecuația: + − = 2019.
Proposed by Ionică Constantin-Romania
VIII.26. Rezolvați în mulțimea numerelor întregi ecuația:
7 + 7 + 8 + 8 + 2018 = 0
Proposed by Ionică Constantin-Romania
a  b b  c c3  a 3
3 3 3 3
VIII.27. Să se demonstreze că    6 , unde a, b și c sunt strict pozitive
ab bc ca
și a  b  c  3 . Când are loc egalitatea? Proposed by Carmen Chirfot-Romania

VIII.28. Să se rezolve ecuația a 2  3  b2  3  c 2  3  6 , unde a, b și c sunt pozitive și


a  b  c  3. Proposed by Carmen Chirfot-Romania

1 1 1
VIII.29. Să se rezolve ecuația a  2 a  b  2 b  c  2 c     12 , unde a, b
a b c
și c sunt strict pozitive și a  b  c  3 . Proposed by Carmen Chirfot-Romania

All solutions for proposed problems can be finded on the


http//:www.ssmrmh.ro which is the adress of Romanian Mathematical
Magazine-Interactive Journal.
9-CLASS-STANDARD

IX.1. If , ∈ ℕ; , , > 0 then:


1 1 1 1
( + 1) + +( + 1) + + + + ≥
( + ) ( + ) ( + ) ( + )
9( + 1)( + 1)

4( + + )
Proposed by D.M. Bătinețu – Giurgiu, Daniel Sitaru-Romania
IX.2. If , ∈ ℕ; , > 0 then in Δ the following relationship holds:
( + )( + ( ) )+( +( ) )( + ( ) )+
+( + ( ) )( + ) ≥ ( + 1)( + 1) √3
Proposed by D.M. Bătinețu – Giurgiu, Daniel Sitaru-Romania

53 ROMANIAN MATHEMATICAL MAGAZINE NR. 22


Romanian Mathematical Society-Mehedinți Branch 2019

IX.3. Let ∈ 0; and ∈ ℕ∗ . Prove:


sin cos
+ ≥4
cos sin
Proposed by Nguyen Van Nho-Vietnam
IX.4. In Δ , , , - cevians, ∈( ), ∈ ( ), ∈ ( )

, , Γ , Γ – inradii, respectively circumradii in Δ ,Δ


, , Γ , Γ – inradii, respectively circumradii in Δ ,Δ
, , Γ , Γ – inradii, respectively circumradii in Δ ,Δ
Prove that:
Γ ⋅ Γ ⋅ Γ ⋅ Γ ⋅ Γ ⋅ Γ ≥ 27 ⋅ ⋅ ⋅ ⋅ ⋅ ⋅
Proposed by Marian Ursărescu – Romania
IX.5. Prove that in any Δ the following relationship holds:

min tan , tan , tan ≤
≤ max tan , tan , tan
2 2 2 + 2 2 2
Proposed by Marian Ursărescu – Romania
IX.6. Let , , , > 0, with + + + = 4. Prove that:
1 1 1 1 1 1 1 1 1 1
+ + + +4 + + + + + ≥
+ + + + + +
1 1 1 1
≥4+6 + + +
+1 +1 +1 +1
Proposed by Andrei Ștefan Mihalcea-Romania
IX.7. Let , , ∈ [0; +∞) ∧ ∑ ( , , ) = 1. Prove:

≤ 3+ ( − 1)
( , , ) ( , , )
Proposed by Nguyen Van Nho- Vietnam
IX.8. Let , , ∈ (0; +∞) ∧ + + ≤ 3. Prove:

1+ + 1+ + 1 + ≤ 3√2
Proposed by Nguyen Van Nho- Vietnam
IX.9. Let , , ∈ [0; +∞) ∧ + + = 3. Prove:

+ + ≤ 81 +
Proposed by Nguyen Van Nho- Vietnam
IX.10. Let , , ∈ [0; +∞) ∧ ∑ ( , , ) = 1. Prove:

+ 3 ≤ 2√3 + 2
( , , )
Proposed by Nguyen Van Nho- Vietnam

IX.11. If , ∈ [0; +∞) and ∈ℕ ∧ ≥ 2 then:
∑ +
( ) ≤ ≤
+1 2
Proposed by Nguyen Van Nho-Vietnam
IX.12. Prove that in any Δ the following inequality holds:

54 ROMANIAN MATHEMATICAL MAGAZINE NR. 22


Romanian Mathematical Society-Mehedinți Branch 2019

tan tan tan


+ + ≥ 3 tan + tan + tan
tan tan tan 2 2 2
Proposed by Marian Ursărescu – Romania
IX.13. Let be Δ , the circumcenter and , , the circumradii of Δ ,Δ and

Δ . Prove that: sin + sin + sin ≤

Proposed by Marian Ursărescu – Romania


IX.14. Prove that in any Δ the following inequality holds:

+ + ≥4−2
Proposed by Marian Ursărescu – Romania
IX.15. In Δ the following relationship holds:

6 ℎ ℎ ℎ
≤ + +
++ +
Proposed by Seyran Ibrahimov-Azerbaijan
IX.16. If , > 0 then in Δ the following relationship holds:
ℎ ℎ ℎ 9
+ + ≥
ℎ ℎ ( ℎ + ℎ ) ℎ ℎ ( ℎ + ℎ ) ℎ ℎ ( ℎ + ℎ ) ( + )
Proposed by D.M. Bătinețu – Giurgiu; Claudia Nănuți – Romania

IX.17. If , , ∈ 0, then in Δ the following relationship holds:


(sin + tan ) (sin + tan ) (sin + tan )
+ + ≥ 16√3
sin sin sin sin sin sin
Proposed by D.M. Bătinețu – Giurgiu; Claudia Nănuți – Romania

IX.18. In Δ the following relationship holds: ( + )ℎ + ( + )ℎ + ( + )ℎ ≥ 12

Proposed by Seyran Ibrahimov- Azerbaijan


IX.19. In Δ the following relationship holds:

2 ⋅ sin ≥ (sin + sin ) + (sin + sin ) + (sin + sin )


Proposed by Seyran Ibrahimov- Azerbaijan
IX.20. In Δ the following relationship holds: + + + + + ≥ 24

Proposed by Seyran Ibrahimov-Azerbaijan


 2
IX.21. Prove that: cos ec  4 cos 2
14 7

Proposed by Vasile Mircea Popa-Romania


IX.22. Să se demonstreze egalitatea:

55 ROMANIAN MATHEMATICAL MAGAZINE NR. 22


Romanian Mathematical Society-Mehedinți Branch 2019

  5   2 3   4 6 
3 2 cos  cos   3 2 cos  cos   3 2 cos  cos   3 7  33 13
 13 13   13 13   13 13 

Proposed by Vasile Mircea Popa-Romania


IX.23. In any Δ the following inequality holds:
1 1 1 8
+ + >
7
Proposed by Marian Ursărescu – Romania
IX.24. Let , , be nonnegative real numbrs such that + + = 3. Prove:
1 1 1 6
+ + ≥
2 +3 2 +3 2 +3 +9
Proposed by Le Khanh Sy-Vietnam
IX.25. Prove that:
2 3 √13 1 5 1
cos cos = cos cos +
13 13 6 3 2√13 12
Proposed by Vasile Mircea Popa – Romania
IX.26. Prove that in any acute-angled triangle the following inequality holds:
1
min(sin sin cos , sin sin cos , sin sin sin ) ≤ + ≤
2
≤ max(sin sin cos , sin sin cos , sin sin cos )
Proposed by Marian Ursărescu – Romania
IX.27. Let be an acute-angled triangle and circumcenter. We denote with , ,
and , , the inscribed circumradii in Δ ,Δ ,Δ . Prove that:
1 1 1 16 √3 + 2
+ + ≥
Proposed by Marian Ursărescu – Romania
IX.28. Solve for real numbers:
tan + tan = − tan
tan + tan = − tan
tan + tan = − tan
Proposed by Seyran Ibrahimov-Maasilli-Azerbaijan
IX.29. If , , ∈ ℝ ∧ + + = , ∈ ℕ. Show:

+ + + + √ + ≤ 3 1+ ⋅3
Proposed by Seyran Ibrahimov-Maasilli-Azerbaijan
IX.30. Let Δ be acute-angled and , , the points in which the heights of the
triangle intersects the circumcenter. If = , then Δ is equilateral.
Proposed by Marian Ursărescu – Romania
IX.31. Prove that in any Δ the following inequality holds:
( + )( + ) ( + )( + ) ( + )( + )
+ + ≤4 +2 +3
Proposed by Marian Ursărescu – Romania
IX.32. In any Δ the following inequality holds:
56 ROMANIAN MATHEMATICAL MAGAZINE NR. 22
Romanian Mathematical Society-Mehedinți Branch 2019
9
+ + ≤ ⋅
4
Proposed by Marian Ursărescu – Romania
IX.33. Prove that in any acute-angled Δ the following inequality holds:

max cot , cot , cot ≥7 −


2 2 2 2
Proposed by Marian Ursărescu – Romania
IX.34. In Δ the following relationship holds:

2( + + )≤(+ + )(2 − max ( , , ))


Proposed by Seyran Ibrahimov- Azerbaijan
IX.35. Let , , be positive real numbers such that: + + = 3. Prove that:
1 1 1
+ + + + + ≥4
+ + + + + +
Proposed by Hoang Le Nhat Tung –Vietnam
IX.36. Let , , be positive real numbers such that: + + =3 . Find the
maximum value of:

( + ) ( + ) ( + )
= + +
− + +1 − + +1 − + +1
Proposed by Hoang Le Nhat Tung –Vietnam

IX.37. Let be the centroid in Δ and ∈ ; ∈ ; ∈ . Prove that in Δ


the following relationship holds:
( + ) ( + ) ( + )
+ + ≥ 4√3
⋅ℎ ⋅ℎ ⋅ℎ
Proposed by D.M. Bătinețu – Giurgiu, Dan Nănuți-Romania
IX.38. Let Δ be the circumcevian triangle of incentre of Δ and , > 0. Prove that:
9
+ + ≥
ℎ ℎ ( ℎ + ℎ ) ℎ ℎ ( ℎ + ℎ ) ℎ ℎ ( ℎ + ℎ ) ( + )
Proposed by D.M. Bătinețu – Giurgiu, Claudia Nănuți-Romania
IX.39. If , ∈ ℕ; ∈ ; ∈ ; ∈ then in Δ the following relationship holds:
+ +
+ ⋅ + + + ⋅
ℎ ℎ
+
⋅ + + + ⋅ + ≥

≥ 4√3( + 1)( + 1)
Proposed by D.M. Bătinețu – Giurgiu, Daniel Sitaru-Romania
IX.40. If , , , ∈ ℕ − {0}, > > > then:
(2 − 1)(2 − 1) > (2 − 1)(2 − 1)
Proposed by Daniel Sitaru – Romania
IX.41. Let , , > 0. Prove that:
+ + + 2 +2 +4 2 +4 +2
+ + ≥ +
+ + 3 + 3 +

57 ROMANIAN MATHEMATICAL MAGAZINE NR. 22


Romanian Mathematical Society-Mehedinți Branch 2019
Proposed by Andrei Ștefan Mihalcea-Romania

IX.42. If , , ∈ 0, then in Δ the following relationshp holds:


sin + tan sin + tan sin + tan
+ + > 4√3
+ + +
Proposed by D.M. Bătinețu-Giurgiu, Neculai Stanciu – Romania
IX.43. If , ∈ ℕ then in Δ the following relationship holds:
( +( ) )( + 1) + ( + )( + ) + ( + 1)( + ( ) ) ≥
≥ 4√3( + 1)( + 1)
Proposed by D.M. Bătinețu – Giurgiu, Daniel Sitaru-Romania
IX.44. Let , , > 0. Show that:
+ + 2 2 1
+ ≥ 2( + + 2 ) + −
+3 3 + +
Proposed by Andrei Ștefan Mihalcea-Romania
IX.45. In Δ the following relationship holds:
2 2
+ + ≥ +

Proposed by Bogdan Fustei-Romania


IX.46. In ∆ the following relationship holds:

+ +
≤ 2 (ℎ + ℎ + ℎ )

Proposed by Bogdan Fustei-Romania
IX.47. In ∆ the following relationship holds:
√3 (ℎ + ℎ + ℎ )( + + )

4 sin sin ( + + ) ℎ
Proposed by Bogdan Fustei-Romania
IX.48. In ∆ the following relationship holds:
+ + ≥ 3 where , , circumradii of Δ ,Δ ,Δ

Proposed by Bogdan Fustei-Romania


IX.49. In ∆ the following relationship holds:
(ℎ + ℎ + ℎ )( + + )
+ + ≥
+ +
where , , circumradii Δ ,Δ ,Δ . Proposed by Bogdan Fustei-Romania
IX.50. In ∆ the following relationship holds:
+ + ≥ 3 where , , circumradii of Δ ,Δ ,Δ
Proposed by Bogdan Fustei-Romania
IX.51. In ∆ the following relationship holds:

+ + ≥ 3 where , , circumradii of Δ ,Δ ,Δ

58 ROMANIAN MATHEMATICAL MAGAZINE NR. 22


Romanian Mathematical Society-Mehedinți Branch 2019
Proposed by Bogdan Fustei-Romania
IX.52. In ∆ the following relationship holds:
+ +
≤ + +
2
Proposed by Bogdan Fustei-Romania
IX.53. In ∆ the following relationship holds:
+ +
>
2 ( − )( − )
Proposed by Bogdan Fustei-Romania
IX.54. In Δ the following relationship holds:

ℎ ℎ ℎ
3( 2 + )≥ + +
Proposed by Seyran Ibrahimov- Azerbaijan
All solutions for proposed problems can be finded on the
http//:www.ssmrmh.ro which is the adress of Romanian Mathematical
Magazine-Interactive Journal.
10-CLASS-STANDARD

X.1. 1. If , , > 0 then in Δ the following relationship holds:


( + ) ( + ) ( + )
+ + ≥ 4√3
ℎ ℎ ℎ
Proposed by D.M. Bătinețu – Giurgiu, Claudia Nănuți-Romania

X.2. If , > 0; – centroid; = ( , ); = ( , ); = ( , ) then in Δ


the following relationship holds:
243
+ + ≥
( + ) ( + ) ( + ) ( + )
Proposed by D.M. Bătinețu – Giurgiu, Daniel Sitaru-Romania

X.3. In Δ , – centroid; = ( , ); = ( , );
). Prove that: = ( ,
81
+ + ≥
( + ) ( + ) ( + ) ( + )
Proposed by D.M. Bătinețu – Giurgiu, Dan Nănuți-Romania
X.4. If , , , ∈ ℕ; , , > 0 then:
( + 1)( + ) +( + )( + )+ +( ) ( + 1) ⋅

59 ROMANIAN MATHEMATICAL MAGAZINE NR. 22


Romanian Mathematical Society-Mehedinți Branch 2019

⋅ ( + )( + )+( + )( + ) + ( + 1)( + ) ≥
≥ ( +1 )( +1 )( +1 )( +1 ) ( + + )
Proposed by D.M. Bătinețu – Giurgiu, Daniel Sitaru-Romania

X.5. In Δ ; – centroid; = ( , ); = ( , ); = ( , ), ∈ ;
∈ ; ∈ . Prove that:
81
+ + ≥
( + ) ( + ) ( + ) ( + )
Proposed by D.M. Bătinețu – Giurgiu, Dan Nănuți-Romania

X.6. If ≥ 0; , , > 0; , ∈ ℕ then:

1 1
(( + ) + + ( + ) + +
2 + 3√ 2 + 3√

1 3( + 1) ( + 1)
+ ( + ) + ≥
2 + 3√ 5
Proposed by D.M. Bătinețu – Giurgiu, Daniel Sitaru-Romania

X.7. In Δ the following relationship holds:


( + + )( + + ) ≥ 27√3 ; = [ ]
Proposed by D.M. Bătinețu – Giurgiu, Daniel Sitaru-Romania

X.8. If , , ∈ 0, then in Δ the following relationship holds:


sin + tan sin + tan sin + tan
⋅ + ⋅ + > 4√3
sin + sin sin + sin sin + sin
Proposed by D.M. Bătinețu – Giurgiu, Daniel Sitaru-Romania

X.9. If , > 0; Δ – circumcevian triangle of incentre in Δ ; – centroid then in


Δ the following relationship holds:
3
+ + ≥
ℎ ℎ ( ℎ + ℎ ) ℎ ℎ ( ℎ + ℎ ) ℎ ℎ ( ℎ + ℎ ) ( + )
Proposed by D.M. Bătinețu – Giurgiu, Dan Nănuți-Romania
X.10. In Δ the following relationship holds:
( + ) ( + ) ( + )
+ + ≥ 4( + 1)√3 − 3 ; ∈ ℕ
ℎ ℎ ℎ
Proposed by D.M. Bătinețu – Giurgiu; Neculai Stanciu – Romania

X.11. If , , ∈ 0, then in Δ the following relationship holds:


(sin + tan ) (sin + tan ) (sin + tan )
+ + > 16√3
sin sin sin sin sin sin
Proposed by D.M. Bătinețu – Giurgiu, Claudia Nănuți-Romania

X.12. In Δ the following relationship holds:


60 ROMANIAN MATHEMATICAL MAGAZINE NR. 22
Romanian Mathematical Society-Mehedinți Branch 2019

√ √2
⋅ ℎ ℎ −2 ≥ +
3
Proposed by Bogdan Fustei – Romania
X.13 In Δ the following relationship holds:
6√3
≤ +
+ +
Proposed by Seyran Ibrahimov- Azerbaijan
X.14. Solve for real numbers:
tan + tan = − tan
tan + tan = − tan
tan + tan = − tan
Proposed by Seyran Ibrahimov- Azerbaijan

X.15. Solve for real numbers: ≠ ∧ ∈ ℝ, − =


=
Proposed by Urfan Aliyev-Azerbaijan
X.16. Prove that in any Δ the following inequality holds:

(( − ) +( − ) +( − ) )
−2 ≥
3
Proposed by Adil Abdullayev- Azerbaijan
X.17. Prove that in any Δ the following inequality holds:
( + + + + + )(ℎ + ℎ + ℎ ) ∑

ℎ + + +ℎ +ℎ −3
Proposed by Bogdan Fustei-Romania
X.18. Let ∈ ℝ. Solve in ℝ:

√1 + + √1 + 2 + √1 + 3 + √1 + 4 + √1 + 5 = 5
Proposed by Nguyen Van Nho- Vietnam

X.19. Let , , ∈ [0; +∞). Prove: (1 + )(1 + )(1 + )≤

Proposed by Nguyen Van Nho- Vietnam


X.20. Let , , ∈ [1; 2] ∧ + + = 6. Prove: + + ≤ 18

Proposed by Nguyen Van Nho- Vietnam


X.21. Let ∈ℕ∧ ≥ 2. Solve in ℝ:

∑ log (2 − )
=1
∑ √1 +
Proposed by Nguyen Van Nho- Vietnam
X.22. Let , , ∈ (0; +∞) ∧ ∑ ( , , ) = 3. Prove:

61 ROMANIAN MATHEMATICAL MAGAZINE NR. 22


Romanian Mathematical Society-Mehedinți Branch 2019

≤3
( , , )
Proposed by Nguyen Van Nho- Vietnam
X.23. Prove that in any Δ the following inequality holds:

ℎ ≤ ℎ −2 ≤ ℎ
Proposed by Bogdan Fustei-Romania
X.24. Prove that in any Δ the following inequality holds:

1 1 1
≥ 3 + +
ℎ ℎ ℎ
Proposed by Bogdan Fustei-Romania
X.25. Prove that in any Δ the following inequality holds:

+ + 1
≥ +
2 2 ℎ
Proposed by Bogdan Fustei-Romania
X.26. Prove that in any Δ the following inequality holds:
2
≥ 1+

Proposed by Bogdan Fustei-Romania


X.27. In all Δ prove:
(4 + ) − 12 3
> 3√3 −
2
Proposed by Rovsen Pirguliyev-Azerbaijan
X.28. Prove that in any Δ the following inequality holds:

− + + + + +
≥ 3√2
ℎ ℎ + + +ℎ +ℎ +ℎ −3
Proposed by Bogdan Fustei-Romania
X.29. Let be any tetrahedron , , , , the exradii of the spheres and = inradii
of the sphere. Prove that:
a) + + + ≤
√ √
b) + + + √ ≥

Proposed by Marian Ursărescu – Romania
X.30. Prove that in any Δ the following inequality holds:

+ + ≥4
sin + sin sin + sin sin + sin
Proposed by Marian Ursărescu – Romania

62 ROMANIAN MATHEMATICAL MAGAZINE NR. 22


Romanian Mathematical Society-Mehedinți Branch 2019

X.31. Prove that in any Δ the following inequality holds:


+ + + + + + +

+ + +ℎ +ℎ +ℎ −3 2∑ℎ − ∑
Proposed by Bogdan Fustei-Romania
X.32. Prove that in any Δ the following inequality holds:
2
≥ 2 −1

Proposed by Bogdan Fustei-Romania


X.33. Prove that in any Δ the following inequality holds:

+ ℎ −3 ≥ + ℎ ℎ ℎ

Proposed by Bogdan Fustei-Romania


X.34. Let be Δ , , , excentres. Prove that:

9 9
≤ + + ≤
2 4
Proposed by Marian Ursărescu – Romania

X.35. Prove that in any acute-angled triangle the following inequality holds:
min{( + ) cot , ( + ) cot , ( + ) cot } ≤ 2 ≤ max{( + ) cot , ( + ) cot , ( + ) cot }
Proposed by Marian Ursărescu – Romania

X.36. Prove that in any Δ the following inequality holds:



min , , ≤ ≤ max , ,
+ + + +2 + + +
Proposed by Marian Ursărescu – Romania

X.37. Let be a tetrahedron, the centroid and any point in space. Prove that:
2
≥ ( ∙ )
3
Proposed by Marian Ursărescu – Romania
X.38. Prove that in any Δ the following inequality holds:
+ +

2 ℎ +ℎ +ℎ
Proposed by Bogdan Fustei-Romania
X.39. Prove that in any Δ the following inequality holds:
+ +
≤ 2
+ +
Proposed by Bogdan Fustei-Romania
X.40. Prove that in any Δ the following inequality holds:

63 ROMANIAN MATHEMATICAL MAGAZINE NR. 22


Romanian Mathematical Society-Mehedinți Branch 2019

⋅ ≥
2 ℎ
Proposed by Bogdan Fustei-Romania
X.41. Let be a tetrahedron and , , , the excenters of the sphere of the
tetrahedron. (centrele sferelor exinscrise tetraedrului). Prove that:
+ + + ≥2
Proposed by Marian Ursărescu – Romania

X.42. Prove that in any acute-angled triangle the following relationships holds:
a) ≥ 18 min cot , cot , cot
b) ≤ max cot , cot , cot
Proposed by Marian Ursărescu – Romania

X.43. Prove that in any acute-angled Δ the following relationship holds:


ℎ + ℎ + ℎ ≤ 3( + )
Proposed by Marian Ursărescu – Romania
X.44. Let be Δ , – incentre and , , – circumradii of Δ ,Δ ,Δ . Prove
that:

2 −2 − ≤ + + ≤4 −8 +3
2 2 2
Proposed by Marian Ursărescu – Romania

X.45. Prove that in any Δ the following inequality holds:


sin + sin 2( + )
≤ ,[ ]= Δ
sin + sin [ ]
Proposed by Marian Ursărescu – Romania

X.46. Δ (sides , , ,),Δ (sides , , ) . Prove that:


2
≥ + +
3
Proposed by Bogdan Fustei-Romania

X.47. Does there exists non-constant functions : (1; +∞) → ℝ such that:

( )= ( )+ (
)+ ⋯+ ( ) ?
Proposed by Rovsen Pirguliyev-Azerbaijan
X.48. In ∆ the following relationship holds:

ℎ 4 8
(ℎ + ) ≥ +
2 3 3

Proposed by Bogdan Fustei-Romania

64 ROMANIAN MATHEMATICAL MAGAZINE NR. 22


Romanian Mathematical Society-Mehedinți Branch 2019

X.49. In ∆ the following relationship holds:

+ + ≤ + +
Proposed by Bogdan Fustei-Romania
X.50. In ∆ the following relationship holds:
ℎ +ℎ
+ + ≥ ⋅ sin
ℎ ℎ ℎ 2 2
Proposed by Bogdan Fustei-Romania
X.51. In ∆ the following relationship holds:

ℎ ℎ ℎ (ℎ − 2 )(ℎ − 2 )(ℎ − 2 )

Proposed by Bogdan Fustei-Romania
X.52. In ∆ the following relationship holds:

ℎ +ℎ ℎ +ℎ ℎ +ℎ
+ + ≥ + +
2 ℎ ℎ ℎ
Proposed by Bogdan Fustei-Romania
X.53. In ∆ the following relationship holds:

√2 4 + ≥
ℎ −2
Proposed by Bogdan Fustei-Romania
X.54. In ∆ the following relationship holds:
1
+ + ≥ sin
2 2
Proposed by Bogdan Fustei-Romania
X.55. In ∆ the following relationship holds:
ℎ ℎ ℎ ℎ ℎ ℎ
+ + ≥ + +

Proposed by Bogdan Fustei-Romania


X.56. In ∆ the following relationship holds:

2+ > + +
2 ℎ ℎ ℎ
Proposed by Bogdan Fustei-Romania

X.57. In ∆ the following relationship holds: + + ≥ √3, , , –


circumradii (razele cercurilor circumscrise), Δ ,Δ ,Δ .
Proposed by Bogdan Fustei-Romania

X.58. Prove that in any triangle the following inequality holds:

65 ROMANIAN MATHEMATICAL MAGAZINE NR. 22


Romanian Mathematical Society-Mehedinți Branch 2019

ℎ 2
≥ + 2
− 3√27
Proposed by Bogdan Fustei – Romania
X.59. Fie n ≥ 3 și z1 , z2 , … , zn ∈ ℂ astfel încât

|S| = |S-z1| + | S – z2 |+ | S – z3 | + … + | S – zn | , unde S = z1 +z2 + … + zn .

Proposed by Butaru Zizi, Bețiu Anicuța-Romania

X.60. Prove that in any triangle the following inequality holds:

+ + ≥6
Proposed by Seyran Ibrahimov –Azerbaijan

X.61. Let , , , be strictly positive real numbers such that + + + = 1. Prove that:

1
+ + + + + ≤
1+ + 1+ + 1+ + 1+ + 1+ + 1+ + 4
Proposed by Vasile Mircea Popa-Romania

X.62. Prove that the following is a perfect square number.


3 4 5 99 100
Find the smallest value of such that above number is perfect square number.
Proposed by Naren Bhandari-Nepal
X.63. In  ABC the following relationship holds:

1 1  4R 
3 p   ra2     p   5 .
b c  r 
Proposed by Marin Chirciu-Romania
X.64. In  ABC the following relationship holds:
2
 2r  1 1
3 p     ha2     3 p .
R b c
Proposed by Marin Chirciu-Romania
X.65. In  ABC the following relationship holds:

2
18r  1 1
  ha2     9 .
R b c

Proposed by Marin Chirciu-Romania


X.66. In  ABC the following relationship holds:

66 ROMANIAN MATHEMATICAL MAGAZINE NR. 22


Romanian Mathematical Society-Mehedinți Branch 2019
2
 1 1  9R
 ra2  b  c   2r .
Proposed by Marin Chirciu-Romania

All solutions for proposed problems can be finded on the


http//:www.ssmrmh.ro which is the adress of Romanian Mathematical
Magazine-Interactive Journal.

11-CLASS-STANDARD

XI.1. Let be : 0, → (0, ∞); ( ) = tan + sin and , , ∈ 0, . Prove that in Δ


the following relationship holds:
( )⋅ ( ) ( )⋅ ( ) ( )⋅ ( )
⋅ + ⋅ + ⋅ ≥ 16√3
sin sin sin
Proposed by D.M. Bătinețu – Giurgiu, Dan Nănuți-Romania

XI.2. If , , > 0; < ; ∈ ℕ; ≥ 2; , ∈ (0, ∞); ≤ ≤ + ; ∈ 1, ; > 0;


∈ 1, then:

+ ( + ) ≤( + + ) ; ∈ ℕ∗

Proposed by D.M. Bătinețu – Giurgiu, Daniel Sitaru-Romania

XI.3. If , , > 0; : (0, ∞) → (0, ∞); ( ) = √ then:


( ) ( ) ( )
+ + > 8√3
Proposed by D.M. Bătinețu – Giurgiu, Claudia Nănuți-Romania

XI.4 Let be : (0, +∞) → ℝ, ( ) = + ln , ∈ ℕ∗ and the sequence


= 1+ , ∈ ℕ∗ . Find: lim → .
Proposed by Marian Ursărescu – Romania

XI.5. Let be , ∈ (ℝ) such that 5 − √5( + ) = 3 + √5( − ) and


− is invertible. Prove that: ⋮ 5. Proposed by Marian Ursărescu – Romania

XI.6. Let be > 0 and = 1+ − 1, ∈ ℕ, ≥ 2. Find: lim →

67 ROMANIAN MATHEMATICAL MAGAZINE NR. 22


Romanian Mathematical Society-Mehedinți Branch 2019
Proposed by Marian Ursărescu – Romania
XI.7. Find:
1
lim ( − 1) 2 − ( − 1)(2 − 1)

Proposed by Marian Ursărescu – Romania


XI.8. Find:
arcsin arcsin ( )
lim − , ∈ ℕ∗
→ ( + 1)
Proposed by Marian Ursărescu – Romania

XI.9. Let be ∈ , (ℂ) and ∈ , (ℂ) such that:


1 1 1
= 0 1 1
0 0 1
Find: ( )+( ) .
Proposed by Marian Ursărescu – Romania
XI.10.

Ω ( )= ( − + 1)( + )!, , ∈ℕ

Find:

Ω = lim Ω ( ) − ( + 1)!

Proposed by Daniel Sitaru -Romania

XI.11. Let be =1+ + +⋯+( )


. Find:

lim −
→ 90
Proposed by Marian Ursărescu – Romania

XI.12. Let be , ∈ (ℝ) such that: ( )= ⋅ . Prove that:


1
det( + ) + det + ≥ (det + det ) , ∀ ∈ ℝ∗
Proposed by Marian Ursărescu – Romania

XI.13. Let be ∈ (ℝ) invertible, such that det( + ) = 0. Prove that: =

Proposed by Marian Ursărescu – Romania


XI.14. Find , , ≥ 0 such that:

= + +2
= + +2

68 ROMANIAN MATHEMATICAL MAGAZINE NR. 22


Romanian Mathematical Society-Mehedinți Branch 2019
= + +2
Proposed by Seyran Ibrahimov- Azerbaijan
XI.15. If 0 ≤ , , < 1 solve the system:

arcsin + arcsin = arctan


arcsin + arcsin = arctan
arcsin + arcsin = arctan
Proposed by Seyran Ibrahimov- Azerbaijan
XI.16. Let f , g : R *  R * be functions such that:

g ( x  1) f ( x)
lim  f ( x  1)  f ( x)   a  R* , lim  b  R* and there exists lim , and
x  x xg ( x) x  x
1

lim
 g ( x)  x 
. For t  R calculate the limit lim  f ( x) 
sin t
cos 2 t
2
sin t 
  g ( x) x 1   g ( x)  x  .
2

x x x
 
Proposed by D.M. Bătinețu – Giurgiu, N. Stanciu – Romania

g ( x  1)
XI.17. Let f , g : R*  R * such that: lim  f ( x  1)  f ( x)   a  R* , lim  b  R* and
x x  xg ( x )

there is lim
f ( x)
, lim
g ( x)  x . For t  R , calculate:
x  x x x
2 2
sin 2 t
cos t cos t 
lim  f ( x)    g ( x) x 1   g ( x)  x  .
x
 
Proposed by D.M. Bătinețu – Giurgiu, Neculai Stanciu – Romania

XI.18. Let be the sequence > 0, = + +⋯+ , ∈ ℕ∗ . Find:



Ω = lim
→ √
Proposed by Marian Ursărescu – Romania

XI.19. Let be the sequence > 0 and = , ∀ ∈ ℕ, ∈ ℕ∗ , ≥ 2. Find:

lim

Proposed by Marian Ursărescu – Romania
n 1
(n  2)
XI.20. Fie g n 
(n  1) n n 
2
, n  N şi x  R . Să se calculeze lim n sin x g ncos
1
2
x
 2
 g ncos x 
Proposed by D.M. Bătinețu – Giurgiu, Neculai Stanciu – Romania

69 ROMANIAN MATHEMATICAL MAGAZINE NR. 22


Romanian Mathematical Society-Mehedinți Branch 2019

(n  2) n1
XI.21. For an n0 , a n 
(n  1) n
2
, x  ( ,  ) , bn ( x)n1 , bn ( x)  n sin x ancos
1
2
x 2

 a ncos x , find 
lim bn ( x) . Proposed by D.M. Bătinețu – Giurgiu, Neculai Stanciu – Romania
n 

XI.22. Find:

lim

1 ⋅ √2! √3! ⋅ … ⋅ √ !
Proposed by Marian Ursărescu – Romania

(ℝ) such that det( ∗


XI.23. Let be ∈ + ) = 0. Prove that: = 1 + det

Proposed by Marian Ursărescu – Romania

XI.24. Find the continuous functions , , ℎ: ℝ → ℝ with the property:


+ ( ) + ℎ( )
= ,∀ , ∈ ℝ
2 2
Proposed by Marian Ursărescu – Romania

cos 2 x cos 2 x
XI.25. Dacă x  R , atunci şirul Ln ( x) n 2 , Ln ( x)  n sin
2
x 



n 1 (n  1)!    n!
n  este


convergent. Să se calculeze lim Ln ( x) .
n

Proposed by D.M. Bătinețu – Giurgiu, Neculai Stanciu – Romania

XI.26. Let be ∈ (ℝ), invertible such that: det( + ) = 0. Prove that:


∗ + det = 1 + det ⋅ ( ∗)
Proposed by Marian Ursărescu – Romania

XI.27. Fie  x n n1 ,  y n n1 , x n  R* , y n  R astfel încât există lim  x n1  x n   x  R* ,
n 

lim y n  y  R
n 
şi lim ( y n  y )n  z  R * . Să se calculeze : lim  x n1 y  x n y n  .
n  n 

Proposed by D.M. Bătinețu – Giurgiu, Neculai Stanciu – Romania

XI.28. Dacă  x n n 1 , x n  R* , n  N * astfel încât există lim ( xn 1  x n )  x  R* .


n 

Să se calculeze :

lim x n 1 n 1 n  1  x n n n .
n 

Proposed by D.M. Bătinețu – Giurgiu, Neculai Stanciu – Romania

XI.29 Prove that if , ∈ (ℂ) such that: ( − ) =


Proposed by Marian Ursărescu – Romania

70 ROMANIAN MATHEMATICAL MAGAZINE NR. 22


Romanian Mathematical Society-Mehedinți Branch 2019

XI.30. Let be ∈ (ℤ). If det is odd, then: det( − 2 ) ≠ 0, ∀ ∈ ℤ.


Proposed by Marian Ursărescu – Romania

( n  2 ) cos 2 x ( n 1) cos 2 x

n
 2
XI.31. Fie x  R . Să se calculeze: lim  n sin x 

n 1
n 1    n
n 



Proposed by D.M. Bătinețu – Giurgiu, Neculai Stanciu – Romania

( n  2 ) sin 2 x ( n 1) sin 2 x

n

XI.32. Fie x  R . Să se calculeze: lim  n cos x 
2


n 1
n 1    n
n 



Proposed by D.M. Bătinețu – Giurgiu, Neculai Stanciu – Romania


XI.33. Find:
1 1 1
lim arctan + arctan( − 1) + ⋯ + arctan 1 ; ∈ ℕ∗ , ≥ 2
→ √ 2
Proposed by Marian Ursărescu – Romania
XI.34. Let be ∈ ( )
ℤ . Find that:
det( + + ∗ ) + det(− + + ∗ ) + det(− + + ∗ ) + det(− ∗ + + ) ⋮ 12
Proposed by Marian Ursărescu – Romania
XI.35. Find:

+ −1
=
→ → ( + + 1)!

Proposed by Daniel Sitaru – Romania

XI.36.
1
( )= (3 )
→ 3

If a, b, c ∈ 0, then:
4 ( )+ ( )+ ( ) ≤ 3( + + )
Proposed by Daniel Sitaru – Romania

XI.37.

Ω = ⎛ (2x + 3x + 1) ⋅ cos (kx) dx⎞


⎝ ⎠
Find:
= ( − ⋅ )

Proposed by Daniel Sitaru – Romania
XI.38. Find:

71 ROMANIAN MATHEMATICAL MAGAZINE NR. 22


Romanian Mathematical Society-Mehedinți Branch 2019

n + 3n + 2 1 (n − 1) + 3(n − 1) + 2 1 3
Ω= log + log + ⋯ + log
n + 3n 4 (n − 1) + 3(n − 1) n 2
Proposed by Daniel Sitaru – Romania

XI.39. Find:

sin
Ω = lim
→ √k + n
Proposed by Daniel Sitaru – Romania

XI.40. Let be , > 0, ≠ such that lim → = lim → = , > 0. Find:



Ω = lim


Proposed by Marian Ursărescu – Romania
XI.41. Let be the sequences: ∈ (0,1), = 1 − , > 0, = + ,
∀ ∈ ℕ, ∈ ℕ, ≥ 2. Find:
lim ⋅

Proposed by Marian Ursărescu – Romania
XI.42.

2 +2 + −1
( )=
( 2 + 2 + 2) ‼

Find:
= !⋅ ( )

Proposed by Daniel Sitaru – Romania

XI.43. If > 0 then:


( )
1 1 ( ) ( )
1 1
+ + > + +
1+ 1+ 1+ 1+
Proposed by Daniel Sitaru – Romania

XI.44. Solve the equation:


sin [ ] + cos [ ] = tan [ ]
, where [⋅] denote the integer part.
Proposed by Rovsen Pirguliyev- Azerbaijan
XI.45. For all > 0, prove:
sin − cos −
+ > |sin | + |cos |
√ √
ln ln
Proposed by Rovsen Pirguliyev- Azerbaijan
XI.46. Prove without computer: >

72 ROMANIAN MATHEMATICAL MAGAZINE NR. 22


Romanian Mathematical Society-Mehedinți Branch 2019
Proposed by Rovsen Pirguliyev- Azerbaijan
XI.47. Let be ∈ (ℝ), invertible such that det( + ) = 0. Prove that:

+ det = det ( )
Proposed by Marian Ursărescu – Romania
XI.48 Let be , ∈ (ℝ) such that det = det . Prove that:

det( + )≥ det( + ),∀ , ∈ ℝ


Proposed by Marian Ursărescu – Romania
XI.49. Find:
1 + 2 √2 + 3 √3 + ⋯ + √
lim ,
→ (1 + 3 + ⋯ + (2 − 1) )

, ∈ℕ . Proposed by Marian Ursărescu – Romania
XI.50. Let be , > 0, = − , = + , ∈ ℕ∗ , ≥ 2. Find:
lim ( )

Proposed by Marian Ursărescu – Romania
XI.51. Ω (x) = ∫ dt, n ∈ ℕ, n ≥ 1. Find:

( − )− ( )
=
→ ( − 1)
Proposed by Ahmad Albaw-Jordan
XI.52. Find:


1+
= 1−
→ √
1+

Proposed by Daniel Sitaru – Romania


XI.53. x = 1, x = 3, x = x + 2x , n ≥ 3. Find:

(−1) ( −2 − )
=

Proposed by Daniel Sitaru – Romania
XI.54. Let be the sequences: > 0, = ln(1 + ) , ∀∈ ℕ and > 0,


= . Find: lim → ( ). Proposed by Marian Ursărescu – Romania

XI.55.
4 1 π π
α(x) = − ⋅ cos (3 x) , x ∈ 0, , β(x) = α − x
3 3 2 2

Find:
( ) ( 3 ) ⋅ … ⋅ ( 2 − 1)
=
→ → (2 ) (4 ) ⋅ … ⋅ (2 )

73 ROMANIAN MATHEMATICAL MAGAZINE NR. 22


Romanian Mathematical Society-Mehedinți Branch 2019
Proposed by Daniel Sitaru – Romania
XI.56. , , > 0,2 ( + + ) = 3 + 2, Ω( ) = lim → − .Prove that:

1
Ω( ) ⋅ Ω( ) ⋅ Ω( ) ≤
27
Proposed by Daniel Sitaru – Romania
XI.57. Find:

1 3
= 3 ⋅
3 − −1

Proposed by Daniel Sitaru – Romania


XI.58 Ω( ) = − + 4 ∑ ( )( )( )
, ∈ℝ

( ) ( )
If ∈ (0,1), > 1 then: Ω( ) + Ω( ) < 1 + Ω( ) ⋅ Ω( )

Proposed by Daniel Sitaru – Romania

All solutions for proposed problems can be finded on the


http//:www.ssmrmh.ro which is the adress of Romanian Mathematical
Magazine-Interactive Journal.
12-CLASS-STANDARD

XII.1. Find:

sin(arctan )
ln
√1 +
Proposed by Abdul Mukhtar-Nigeria
XII.2. Find:

cos + sin
sin(2 ) − sin (2 )

Proposed by Abdul Mukhtar-Nigeria


XII.3. Find:

74 ROMANIAN MATHEMATICAL MAGAZINE NR. 22


Romanian Mathematical Society-Mehedinți Branch 2019
2 +5 +6 + 6 + 12
( + 2 + 2) √ +2 +2
Proposed by Abdul Mukhtar-Nigeria
XII.4. Find:

ln(1 − )
+
(4 + 2 + ) 2 −2 +1

Proposed by Abdul Mukhtar-Nigeria

XII.5. Let ∈ ℕ∗ and ∈ ℝ, ∀ = 1; . Find Ω = ∫ ln(∏ ( − )) .


> max |∀ ∈ 1;
Proposed by Nguyen Van Nho-Vietnam

XII.6. Let be : ℝ → ℝ continuous such that ( ) + ( ) + ( ) = − , ∀ ∈ ℝ, >


0. Find: ∫ ( ) . Proposed by Marian Ursărescu – Romania

XII.7. Find:

+2 +3 +4 +5
−7 + 5 + 5 + 9 + 52 − 60
Proposed by Seyran Ibrahimov- Azerbaijan
XII.8. Find:

+3 +5 +7
= , ∈ℝ
7 + 14 − 2 − + 2 − 11 − 6
Proposed by Seyran Ibrahimov- Azerbaijan
XII.9. Find:
+ + +
Ω= , >0
+ + + + + + +
Proposed by Naren Bhandari-Nepal
XII.10. Find:
2 +3 +9 +4
=
+ 2 + 9 + 8 + 16
Proposed by Seyran Ibrahimov- Azerbaijan
XII.11. Find:
+ + +1
, >0
+ 10 + 28 + 28 + 27 + 18
Proposed by Seyran Ibrahimov- Azerbaijan
XII.12. Find:

75 ROMANIAN MATHEMATICAL MAGAZINE NR. 22


Romanian Mathematical Society-Mehedinți Branch 2019

1 3 2
= −
→ 1+3 1+2

Proposed by Daniel Sitaru– Romania


XII.13. Find:

√ ⋅√

=
→ √ ⋅√

Proposed by Daniel Sitaru– Romania
XII.14. Find:

2 +3
=
→ ( + 1)( + 2)( + 3) +
Proposed by Daniel Sitaru– Romania
XII.15. Find:

= ( + 1) ⎛ ⎞
→∞ +
⎝ ⎠
Proposed by Nho Nguyen Van-Vietnam
XII.16. Find:

1 +
=
→ ( + 1) +
Proposed by Daniel Sitaru– Romania
XII.17. Find:

1 ( + 1)(2 + 1)
=
→ ( + 1)(2 + 1) ( + 1)(2 + 1)
Proposed by Daniel Sitaru– Romania
XII.18. Find:

1 3 3 3
= 2+ 2+ 2+

Proposed by Daniel Sitaru – Romania


XII.19.
(1 + 2 + ) (1 − 2 + )
( )= , ∈ ℕ, ≥2
(1 − )(1 + 2 + )

Find:

76 ROMANIAN MATHEMATICAL MAGAZINE NR. 22


Romanian Mathematical Society-Mehedinți Branch 2019
( + 2) − ( + 1)
=
→ ( + 1) − ( )
Proposed by Daniel Sitaru – Romania

XII.20. Find: Ω = lim →

Proposed by Daniel Sitaru– Romania


XII.21. Let : [0,1] → (−1,1) be a continuous function so that ∫ ( ) ∉ {−1,1}.Prove
∫ ( ) ( )
that: ( )
≤∫ ( ) .

Proposed by Soumitra Mandal - India


XII.22. If 0 < < < then:

7 tan sin + tan √sin tan > −


2 2
Proposed by Daniel Sitaru -Romania

XII.23. Let f: ℝ → ℝ be integrable and satisfying

f(xt + (1 − t)y) ≤ tf(x) + (1 − t)f(y) where x, y ∈ ℝ and t ∈ (0,1)


Show that:
1 1
f(x ) dx + f(x ) dx ≥ 2 (ab) f(a −a b +a b −a b + b )dx
ln a ln b
where , > 0.
Proposed by Soumitra Mandal-India
XII.24. If 0 < < < then:

cos 1 sin
< log ⋅ log
sin + 4 cos 5 sin
Proposed by Daniel Sitaru -Romania

XII.25 Find: Ω = lim → ∫ nx + − nx dx, [∗] - great integer function

Proposed by Daniel Sitaru -Romania

XII.26. Find:

√1 + x − 1
Ω = lim tan dx
→ x
Proposed by Abdul Mukhtar-Nigeria

77 ROMANIAN MATHEMATICAL MAGAZINE NR. 22


Romanian Mathematical Society-Mehedinți Branch 2019

XII.27. If 0 < < 1, : , → ℝ, convexe and increasing function then:

1− 1+ 1+ 1− 1
≤ ( ) ≤ ( )+
2 2 2
Proposed by Abdallah El Farissi-Bechar-Algerie

XII.28. If , , ∈ 0, then:

0≤ (tan + 2 tan tan ) + 4 tan ≤

Proposed by Daniel Sitaru – Romania


XII.29. If , , > 0 then:

∑ ∫
≤ ⋅ ⋅
Proposed by Daniel Sitaru-Romania

( )
XII.30. If , , ≥ 0, ( )=∫ then:
( )( )

( )+ ( )+ ( ) ≥ log +1+
Proposed by Daniel Sitaru – Romania

All solutions for proposed problems can be finded on the


http//:www.ssmrmh.ro which is the adress of Romanian Mathematical
Magazine-Interactive Journal.
UNDERGRADUATE PROBLEMS

U.1. An intriguing mock exponential integral with an improper closed-form.

( − 1) tanh ( ) 4
⋅ =− − 3 log √2 − log(7) +
( + 1)

78 ROMANIAN MATHEMATICAL MAGAZINE NR. 22


Romanian Mathematical Society-Mehedinți Branch 2019

Γ Γ
+5(log(3) − log(5)) + log
Γ Γ
– Catalan Constant.
Proposed by Srinivasa Raghava-AIRMC-India
U.2. Application of the Fourier series:

sin (2 + 1) sinh( )
(−1) = ⋅
+ ( 2 + 1) 4 cosh
provided | | ≤ & > 0, >0
sin (2 + 1) tanh
(−1) = ⋅
+ (2 + 1) 2
Proposed by Srinivasa Raghava-AIRMC-India
U.3. Find:

− 2 log
Ω=
+ log (1 + )
Proposed by Khalef Ruhemi-Jarash-Jordan
U.4. Find:
ln +√ +1
√ +1
Proposed by Abdul Mukhtar-Nigeria
U.5. Evaluate the following integral:

sin sin( ) sin(2 )

and prove that


13
sin sin( ) sin (2 ) ≈
20
Proposed by K. Srinivasa Raghava – AIRMC – India
U.6. If we define the function ( ) for any > 1
( )= log ( ) (tanh ( ) − tanh ( ))

Then prove that:
1
lim ( ) = 14 (3) − ( ( − 4) + 8) + 8 log(2) − 16 − 8
→ 2
Proposed by K. Srinivasa Raghava – AIRMC – India
U.7. Find:
⋅ tan
Ω=
+ +1

79 ROMANIAN MATHEMATICAL MAGAZINE NR. 22


Romanian Mathematical Society-Mehedinți Branch 2019
Proposed by Vasile Mircea Popa – Romania
U.8. Prove that:
sin( ) 2 5
2+ ( coth( ) − 1) = +
sinh(2 ) 3 36
Proposed by Srinivasa Raghava-AIRMC-India
U.9. If

(coth( ) + cos( )) = ( )+ (tanh( ) + sin( ))

then we have

√ + −√ = 2( − 1 )

Proposed by Srinivasa Raghava-AIRMC-India
U.10. For sufficiently large positive integer (some times we can say approaches to
infinity), we have:
1
≥ log( ) + 2
( − 1)
here = .57721566 … Euler’s constant
Proposed by Srinivasa Raghava-AIRMC-India
U.11. Show that:

1
+ sin + tanh =
√2 2 2 √
1 √17 − 4
= 2 1 + √2 − 1 +
2 17
Proposed by K. Srinivasa Raghava – AIRMC – India
U.12. Evaluate the integral:

ln 1+ − ln 1− ln( )
and then prove this sharp inequality

ln 1+ − ln 1− ln( ) ≤4
Proposed by Srinivasa Raghava-AIRMC-India
U.13. Prove that:
(tan ( ) + tan ( )) log(tan( )) log(tan( )) (tan( ) tan( ))

tan( ) tan( )
( ) ( 6) −
= − ,
( )
Where is the Catalan’s constant, denotes the Polygamma function, designates the
Riemann zeta function and ( ) represents the inverse tangent integral.
Proposed by Cornel Ioan Vălean – Romania

80 ROMANIAN MATHEMATICAL MAGAZINE NR. 22


Romanian Mathematical Society-Mehedinți Branch 2019

U.14. Evaluate the sum in closed form, for ≠0


1

, ∈ sinh + (2 + 1)
Proposed by Srinivasa Raghava-AIRMC-India
U.15. For any ≥ 1, we have

2 1− cos( ) + log(cos( )) + log(cos ( )) =

√ Γ
= ( − ( + 1) log(2)) −
Γ 1+

2 1− cos( ) + log(cos( )) + log (cos( )) =

√ Γ 1
= − + log(2) (log(2) − 2 ) +
Γ 1+ 2 24
Inspired from Kartick Chandra Betal
Proposed by K. Srinivasa Raghava – AIRMC – India
U.16. Prove that:
1 1 1 1 (−1)
+ + ≥
5 +4 5 +1 5 +3 5 +2 4 +1
where is Euler’s constant.
Proposed by K. Srinivasa Raghava – AIRMC – India
U.17. Calculate the integral:

− +1

It is required to express the integral value with the usual mathematical constants, without
using values of special functions. Proposed by Vasile Mircea Popa-Romania

U.18. Find:

|sin |
Ω = lim

Proposed by Vasile Mircea Popa-Romania

U.19. Prove that:


ℭ ℭ
sin( ) sin ( )
(cos ( ) − cos ( )) = (cos ( ) − cos ( ))
sin( ) sin( )

where {ℭ, , , , } are integers, ℭ < { , } , ≥ ,


ℭ ℭ

81 ROMANIAN MATHEMATICAL MAGAZINE NR. 22


Romanian Mathematical Society-Mehedinți Branch 2019
Proposed by Ahmed Albaw-India
U.20. Calculate the integral:

√ √
ln

2 +

Proposed by Ekpo Samuel-Nigeria


U.21. Calculate the integral:

log( ) sin ( )

Proposed by Zaharia Burghelea-Romania


U.22. Find:
1+
Ω = lim
→ 1−
Proposed by Ekpo Samuel-Nigeria
U.23.
!+ ≤ ≤ !+
Is there any prime number such that given inequality above can be true, where 1 ≤ ≤
and > 2?
Proposed by Naren Bhandari-Nepal
U.24. Prove that for ∈ℤ




=
Setting = = =⋯= = 2 for = 5 also prove that:


√ √

2 = √2 <2
Proposed by Naren Bhandari-Nepal
U.25.
cos =
128

⎛ ⎞
1 + √2 + 2√2 + 2 2√2 + 2 2 2√2 + + 2 2 2 2√2
⎜ ⎟
⎜ ⎟
⎜ ⎟
⎜ ⎟
2 2 2 2 2√2
⎝ ⎠
Proposed by Naren Bhandari-Nepal

82 ROMANIAN MATHEMATICAL MAGAZINE NR. 22


Romanian Mathematical Society-Mehedinți Branch 2019

U.26.
( ) ( ) ( ) ( ) ( )
−1 +3 + 2 (2 + 1 )
+ =
( 2 + 1)
= 60 − , , ∈ ℝ, ( ) – sine integral function
Find:
(−1)
Ω= + + cos
4
Proposed by Arafat Rahman Akib-Bangladesh
U.27.
1
+ = ⋅ ( )
√ √ √

Find: Ω = ℒ ( ) ⋅ log √2 , ℒ - Lapalce’s transform


Proposed by Arafat Rahman Akib-Bangladesh
U.28. Let

( ) = 1 + ( − 1) + 1 (−1)

+1 +1
then for any ≥ 1, ≥ 1, we have
( )= ( ) −
Note: ( ) =
– Harmonic Number
Proposed by Srinivasa Raghava-AIRMC-India
U.29. An Unique Integral
+ +
=3
(log( ) + log( ) + log( ))
Proposed by Srinivasa Raghava-AIRMC-India
U.30. If,
cosh( ) ( )
Φ( ) =
(2 )!
then

( ) =2 ( ) ln( )

where = √−1
Proposed by Srinivasa Raghava-AIRMC-India
U.31. ( , , ) = ∫ sin ; , > 0, = −2, −1,0,1 …
Find:

83 ROMANIAN MATHEMATICAL MAGAZINE NR. 22


Romanian Mathematical Society-Mehedinți Branch 2019

( , , )
Ω = lim lim lim
→ → →

Proposed by Feti Sinani-Kosovo
U.32. Find:
( )

 )
(
)
 1+ )

( (

→ 
 1+ −
where ( ) is Riemann Zeta function and Γ( ) is Gamma function.
Proposed by Feti Sinani-Kosovo
U.33. Calculate the integrals:

1 √1 + −1
ln(1 + tan + tan ) , , tan
ln(1 + tan + tan )

Proposed by Abdul Mukhtar-Nigeria


All solutions for proposed problems can be finded on the
http//:www.ssmrmh.ro which is the adress of Romanian Mathematical
Magazine-Interactive Journal.
ROMANIAN MATHEMATICAL MAGAZINE-R.M.M.-sUMMER 2018

PROBLEMS FOR JUNIORS


JP.136. Let , , be positive real numbers such that: = . Find the maximum of the
expression:

= + +
+ − + + − + + − +
Proposed by Hoang Le Nhat Tung – Hanoi – Vietnam
JP.137. Let , ≥ . Prove that:

84 ROMANIAN MATHEMATICAL MAGAZINE NR. 22


Romanian Mathematical Society-Mehedinți Branch 2019

( − )
+ ≥ +
( + + )( + + )

Proposed by Andrei Ștefan Mihalcea – Romania

JP.138. Let , , > 0, with + + = . Prove that:

( − )( − )( − )≥ √
Proposed by Andrei Ștefan Mihalcea – Romania
JP.139. Let , , be positive real numbers such that: + + = . Find the minimum
of the expression:

= + +
+ + +

Proposed by Hoang Le Nhat Tung – Hanoi – Vietnam


JP.140. Let , , > 0. Prove that:

√ + ∑
≤ −

Proposed by Andrei Ștefan Mihalcea – Romania


JP.141. Let , , > 0. Prove that:

+ (∑ )

Proposed by Andrei Ștefan Mihalcea – Romania


JP.142. Let , , ≥ . Prove that:

− ∑
≤ −

Proposed by Andrei Ștefan Mihalcea – Romania


JP.143. In any triangle the following relationship holds:

+ + ≤ −
⋅ ⋅ ⋅
all notations are usual sense.
Proposed by Mehmet Șahin – Ankara – Turkey
JP.144. In any triangle the following relationship holds:

85 ROMANIAN MATHEMATICAL MAGAZINE NR. 22


Romanian Mathematical Society-Mehedinți Branch 2019

+ + ≥

Proposed by Mehmet Șahin – Ankara – Turkey


JP.145. In any triangle the following relationship holds:

+ + ≤
+ + +
Proposed by Mehmet Șahin – Ankara – Turkey
JP.146. Let , , be positive real numbers such that: = . Find the maximum of the
expression:

= + +
( − + ) ( − + ) ( − + )
Proposed by Hoang Le Nhat Tung – Hanoi – Vietnam
JP.147. Let , , be positive real numbers such that: + + = . Find the
minimum of the expression:

= + +
( + ) ( + ) ( + )
Proposed by Hoang Le Nhat Tung – Hanoi – Vietnam
JP.148. Let , , be positive real numbers such that: + + = . Prove that:
+ + +
+ + ≥
− + − + − +
Proposed by Hoang Le Nhat Tung – Hanoi – Vietnam
JP.149. Find all functions: : ( , +∞) → ℝ which verify the relationship:
( )≤ ( )+ ( )≤ ( ), ∀ , >0
Proposed by Marian Ursărescu – Romania
JP.150. Let be , , ∈ ℂ∗ different in pairs such that | | = | | = | |. If ( +
+ + + = , then , , are the affixes of an equilateral triangle.

Proposed by Marian Ursărescu – Romania


PROBLEMS FOR SENIORS

SP.136. Let , , be positive real numbers such that: + + = + + . Find


the maximum of the expression:

86 ROMANIAN MATHEMATICAL MAGAZINE NR. 22


Romanian Mathematical Society-Mehedinți Branch 2019

+ + +
= + +

Proposed by Hoang Le Nhat Tung – Hanoi – Vietnam


SP.137. Let , , > 0 such that: + + = . Prove that:
√ +√ +√
+ + + ≥
( + )+ ( + )+ ( + )+
Proposed by Hoang Le Nhat Tung – Hanoi – Vietnam
SP.138. Let , , be positive real numbers such that: + + = . Prove that:

+ + ≥
( + ) ( + ) ( + )
Proposed by Hoang Le Nhat Tung – Hanoi – Vietnam
SP.139. In triangle the lengths of sides , , are , , . Let , , be the
distances from , , to , , ; , , are the lengths of the bisectors , , .
Prove that:

+ + ≥ + +

Proposed by Hoang Le Nhat Tung – Hanoi – Vietnam


SP.140. Let , , be positive real numbers. Prove that:
+ + + ( + + ) ( + + )
+ + ≥ +
+ + + +
Proposed by Hoang Le Nhat Tung – Hanoi – Vietnam
SP.141. Let , , > 0 such that: + + = . Prove that:
+ +
+ + ≥
−√ + −√ + −√ +
Proposed by Hoang Le Nhat Tung – Hanoi – Vietnam
SP.142. Let , , be positive real numbers such that: = . Prove that:
+ + +
+ + ≤
− + + − + + − + +
Proposed by Hoang Le Nhat Tung – Hanoi – Vietnam
SP.143. Let , , be non-negative real numbers. Prove that:

+ + + + + ≥ + + + + +
Proposed by Do Quoc Chinh – Ho Chi Minh – Vietnam
87 ROMANIAN MATHEMATICAL MAGAZINE NR. 22
Romanian Mathematical Society-Mehedinți Branch 2019
SP.144. Let , , be the corners in a triangle . Prove that:

+ + ≥

Proposed by Hoang Le Nhat Tung – Hanoi – Vietnam


SP.145. If < ≤ then:
( )
+

+ +

Proposed by Daniel Sitaru – Romania


SP.146. Let be , ∈ (ℝ) such that:

= −

Find: (( ) − ).
Proposed by Marian Ursărescu – Romania
SP.147. Find all continuous functions : ℝ → ℝ having the property:
( )+ ( )+ ( )= + + , ∀ ∈ ℝ.
Proposed by Marian Ursărescu – Romania
SP.148. Let be > 0 and = , ∀ ∈ ℕ. Find: → ⋅ .

Proposed by Marian Ursărescu – Romania

SP.149. Let be the sequence ( ) ∈ℕ : > 1 and = + , ∀ ∈ ℕ. Find

→ .
Proposed by Marian Ursărescu – Romania
SP.150. Let be ∈ ℤ, = + + ⋯+ + , such that
, ,…, ∈ {± , ± , … , ± }. If is a prime number, > then is irreducible
over ℤ.
Proposed by Marian Ursărescu – Romania

UNDERGRADUATE PROBLEMS
UP.136. Prove that:

88 ROMANIAN MATHEMATICAL MAGAZINE NR. 22


Romanian Mathematical Society-Mehedinți Branch 2019

1 2+ ( )
( )=
4 √1 −
where, ( ) and ( ) denotes the Chebyshev Polynomials of first and second kind.
Proposed by Shivam Sharma – New Delhi – India
UP.137. Let , : ℝ∗ → ℝ∗ be functions such that:
( ) ( )
→ ( + )− ( ) = ∈ ℝ∗ , → ( )
= ∈ ℝ∗ and exists → and

( )
→ . For ∈ ℝ calculate the limit:

( ) ( ) − ( )

Proposed by D.M. Bătinețu – Giurgiu, Neculai Stanciu – Romania


UP.138. Let , : ℝ∗ → ℝ∗ such that:
( )
→ ( + )− ( ) = ∈ ℝ∗ , → ( )
= ∈ ℝ∗ and exists

( ) ( )
→ , → .

For ∈ ℝ, calculate:

( ) ( ) − ( )

Proposed by D.M. Bătinețu – Giurgiu, Neculai Stanciu – Romania


UP.139. Calculate:

( + ) − ( + )

where ∈ ℝ and is the Gamma function (Euler integral of the second kind).
Proposed by D.M. Bătinețu – Giurgiu, Neculai Stanciu – Romania
UP.140. Calculate:

( + ) − ( ( + )

where ∈ ℝ and is the Gamma function (Euler integral of the second kind).
Proposed by D.M. Bătinețu – Giurgiu, Neculai Stanciu – Romania

89 ROMANIAN MATHEMATICAL MAGAZINE NR. 22


Romanian Mathematical Society-Mehedinți Branch 2019
( )
UP.141. For { } , = ( )
, ∈ (−∞, ∞), { ( )} ,

( )= − , find → ( )

Proposed by D.M. Bătinețu – Giurgiu, Neculai Stanciu – Romania


UP.142. Let ( ) be a sequence which satisfy:

− ( + )+ =

where is positive integer and is Euler-Mascheroni’s constant. Compute: →

Proposed by D.M. Bătinețu – Giurgiu, Neculai Stanciu – Romania

UP.143. Let , ∈ℝ , ( , )=− ( + )+∑ with → ( , )=

( , ) ∈ ℝ. Calculate:

+ − ( , ) .
→ + +

Proposed by D.M. Bătinețu – Giurgiu, Neculai Stanciu – Romania

UP.144. Let ∈ ( , ∞) and ( ) = + , for any ∈ ( , ∞). Calculate:

( + )− ( ) .
→ →

Proposed by D.M. Bătinețu – Giurgiu, Neculai Stanciu – Romania


UP.145. Let be ( ) , ∈ ℝ∗ , ∀ ∈ ℕ∗ , such that exists → ( − )= ∈ ℝ∗ .
Find:
( + )

→ ( + )‼ ( − )‼
Proposed by D.M. Bătinețu – Giurgiu, Neculai Stanciu – Romania
( )
UP.146. Let : ( , ∞) → ( , ∞) be a function with → = ∈ ( , ∞) and ∈ ℝ. Find:

( + ) ⋅ ( ) ( )… ( ) ( + ) − ⋅ ( ) ( )… ( )

Proposed by D.M. Bătinețu – Giurgiu, Neculai Stanciu – Romania


UP.147. In an triangle let be , , the lengths of , , , and , , exradii.
Prove that:

90 ROMANIAN MATHEMATICAL MAGAZINE NR. 22


Romanian Mathematical Society-Mehedinți Branch 2019
( + + )
+ + ≥

Proposed by Hoang Le Nhat Tung – Hanoi – Vietnam


UP.148. Let , , be positive real numbers such that: + + = . Prove that:

( + + )+ ≥ ( + + )
Proposed by Hoang Le Nhat Tung – Hanoi – Vietnam
UP.149. Prove that:

( + + )! ( )! ( )! ( )!
(− ) =
+ + + ( + )! ( + )! ( + )! ( )! ( )! ( )!

Proposed by Shivam Sharma – New Delhi – India


UP.150. If , , > 0 then:

≥( + + )
+( )
Proposed by Daniel Sitaru – Romania

All solutions for proposed problems can be finded on the


http//:www.ssmrmh.ro which is the adress of Romanian Mathematical
Magazine-Interactive Journal.
ROMANIAN MATHEMATICAL MAGAZINE-R.M.M.-WINTER 2018

PROBLEMS FOR JUNIORS


JP.151. Let , be positive real numbers. Find the maximum of such that inequality is
true:
+
+ + ≥
+ ( + )
Proposed by Hoang Le Nhat Tung – Hanoi – Vietnam

91 ROMANIAN MATHEMATICAL MAGAZINE NR. 22


Romanian Mathematical Society-Mehedinți Branch 2019
JP.152. Let ; , , denote the lengths of the highways from the peaks
, , ; , , are the lengths of the symmetric divergence lines from the peaks
, , ; , , are the radius of the circle next to the corners , , . Prove that:

+ + ≥

Proposed by Hoang Le Nhat Tung – Hanoi – Vietnam


JP.153. In the lengths , , are , , . Let , , are the length of the
bisectors from the vertices , , in triangle . Prove that:

+ + ≤ ( + + )

Proposed by Hoang Le Nhat Tung – Hanoi – Vietnam


JP.154. In triangle with sides = , = , = . , , are exradii,
, , are distances from , , to , , . Prove that:

+ + ≥ + +

Proposed by Hoang Le Nhat Tung – Hanoi – Vietnam


JP.155. Let , , be positive real numbers such that: + + = . Prove that:

+ + ≥
( + ) ( + ) ( + )
Proposed by Hoang Le Nhat Tung – Hanoi – Vietnam
JP.156. Let be a triangle having the area . Let be ∈ such that the incircles of
, ′ have the same radius. Analogous, we obtain the points ∈ , ∈ .
Prove that:
⋅ ⋅
=

Proposed by Marian Ursărescu – Romania


JP.157. Prove that in , the following inequality holds:

⋅ ⋅ ⋅ ≥
( − )

Proposed by Marian Ursărescu – Romania


JP.158. Let , , > 0. Prove that:

92 ROMANIAN MATHEMATICAL MAGAZINE NR. 22


Romanian Mathematical Society-Mehedinți Branch 2019

+ ≥
+ +

Proposed by Andrei Ștefan Mihalcea – Romania


JP.159. Prove that in any the following inequality holds:
+ + ≤ ( + )
Proposed by Marian Ursărescu – Romania
JP.160. Prove that for all non-negative real numbers , , the following inequality holds:
+ + +
+ + ≥ + +
( + ) ( + ) ( + ) + + +
Proposed by Nguyen Viet Hung – Hanoi – Vietnam
JP.161. Let , , , be positive real numbers such that ≥ . Prove that:
+ + + + + + + + +
+ + +
+ + + + + + + + +
+ + +
+ ≤
+ + +
Proposed by Nguyen Viet Hung – Hanoi – Vietnam
JP.162. Let , , be non-negative real numbers such that + + = . Prove that:

(a) ≤ + + ≤ .
√ √

(b) ≤ + + ≤ . When do the equalities occur?


Proposed by Nguyen Viet Hung – Hanoi – Vietnam


JP.163. Let be an acute triangle with standard notations. Prove that:
+ +
+ + ≤

Proposed by Nguyen Viet Hung – Hanoi – Vietnam

JP.164. If , ≥ then: + +√ + ≥ √
Proposed by Daniel Sitaru – Romania
JP.165. If , , ≥ then:

( + + )≤ √ − + + + + +

Proposed by Daniel Sitaru – Romania


PROBLEMS FOR SENIORS

93 ROMANIAN MATHEMATICAL MAGAZINE NR. 22


Romanian Mathematical Society-Mehedinți Branch 2019
SP.151. Let , , be positive real numbers such that: + + = . Prove that:
+ + ≥
( + )+ + )+ ( ( + )+
Proposed by Hoang Le Nhat Tung – Hanoi – Vietnam
SP.152. Let , , be positive real numbers. Find the minimum of value:
= + + + + +
( + ) ( + ) ( + )
Proposed by Hoang Le Nhat Tung – Hanoi – Vietnam
SP.153. Solve the system of equations:

⎧ + = ( + )+

⎨ +
⎪ − + =

Proposed by Hoang Le Nhat Tung – Hanoi – Vietnam
SP.154. Let , , be positive real numbers such that: + + = . Find the minimum of
value:

√ + +√ +
= + ( + + )
√ + +√
Proposed by Hoang Le Nhat Tung – Hanoi – Vietnam
SP.155. Let , , be positive real numbers such that: = . Find the minimum of
value:
+ +
= + + +
( − + ) ( − + ) ( − + )
Proposed by Hoang Le Nhat Tung – Hanoi – Vietnam

SP.156. Find all the polynomials ∈ ℝ[ ] having the property ( ) = +√ + ,


∀ ∈ ℝ.
Proposed by Marian Ursărescu – Romania
SP.157. Let : [ , +∞) → [ , +∞) a derivable function and > 1. If
( )( ( ) + + + ) = ,∀ ≥ then: → ( ) exists and it is finite.
Proposed by Marian Ursărescu – Romania
SP.158. Prove that for any real numbers , ,…, , the inequality holds:

( + ) + … + ≥

94 ROMANIAN MATHEMATICAL MAGAZINE NR. 22


Romanian Mathematical Society-Mehedinți Branch 2019
where is a positive integer and = . When does the equality occur?
Proposed by Nguyen Viet Hung – Hanoi – Vietnam
SP.159. Prove that in any triangle with standard notations, the inequality holds:
( + + )( + + )≥ ( + ) + ( + ) + ( + )
Proposed by Nguyen Viet Hung – Hanoi – Vietnam

SP.160. Let , , be positive real numbers such that + + ≥ + + . Prove that:

( + + )≥( + + )
Proposed by Nguyen Viet Hung – Hanoi – Vietnam
SP.161. Prove that the following inequalities hold for all real numbers , , ∈ [ , ]
(a) ( − + )( − + )( − + )≤( − + ).
(b) ( − + ) ( − + ) ( − + ) ≤
≤( − + )( − + )( − + )
When does the equality occur?
Proposed by Nguyen Viet Hung – Hanoi – Vietnam
SP.162. If ∈ [ , ∞), , , , ∈ ( , ∞), then in any triangle , with usual notations
holds:

+ ( + )
≥ √
+ ( + )

Proposed by D.M. Bătinețu – Giurgiu, Neculai Stanciu – Romania


SP.163. If ∈ [ , ∞), , , , ∈ ( , ∞), then in any triangle , with usual notations
holds:
( + ) ( + )

+ ( + )
Proposed by D.M. Bătinețu – Giurgiu, Neculai Stanciu – Romania
SP.164. If , , > 0 then:

( + ) + − +( + ) + − +( + ) + − ≥
≥ ( + + )
Proposed by Daniel Sitaru – Romania
SP.165. If , , ≥ then:

( + ) + +( + ) + +( + ) + ≥

95 ROMANIAN MATHEMATICAL MAGAZINE NR. 22


Romanian Mathematical Society-Mehedinți Branch 2019

≥ √ − ( + + )
Proposed by Daniel Sitaru – Romania
UNDERGRADUATE PROBLEMS
UP.151. Given real numbers , ,…, ∈ [ , ]. Find the maximum and minimum
possible value of
= + + ⋯+ +( − )( − )…( − )
Proposed by Nguyen Viet Hung – Hanoi – Vietnam
UP.152. Let , ,…, ( ≥ ) be positive real numbers such that:
( − )

Prove that:


+ −

Proposed by Nguyen Viet Hung – Hanoi – Vietnam


UP.153. Find:

= ( )

Proposed by Ekpo Samuel – Nigeria


UP.154. If ∈ ( , ∞ ); ∈ ℕ; ≥ ; → = > 0 then find:

= ⋅ √ !+ ( + )!

[∗] - great integer function.


Proposed by D.M. Bătinețu – Giurgiu, Daniel Sitaru – Romania
UP.155. If ∈ ℕ; , , > 0 then in the following relationship holds:

+ + + ≥ ( + )√

Proposed by D.M. Bătinețu – Giurgiu, Neculai Stanciu – Romania


UP.156. If , , > 0; < ; ( + ) ∈ ℕ; ( + ) > 1; ∈ ℕ; ≥ ; ∈ [ , ]; >
0; ∈ , then:

96 ROMANIAN MATHEMATICAL MAGAZINE NR. 22


Romanian Mathematical Society-Mehedinți Branch 2019
( )
+ +
≤ ⋅
+ ( + )

Proposed by D.M. Bătinețu – Giurgiu, Neculai Stanciu – Romania


UP.157. If ≥ ; , > 0 then in the following relationship holds:

+ + ≥
( + ) ( + ) ( + )

≥ ⋅ ⋅
( + )
Proposed by D.M. Bătinețu – Giurgiu, Neculai Stanciu – Romania
UP.158. Find:

= ( )

Proposed by Arafat Rahman Akib – Dhaka – Bangladesh


UP.159. Find:

= ( + + ) ( − )

Proposed by Ekpo Samuel – Nigeria


UP.160. Let , , ∈ ( ; +∞) and ≥ . Prove:

+ + ≥
([ ] + { } + ) ([ ] + { } + ) ([ ] + { } + ) ( + + + )
[ ] is the integer part of the real number ;
{ } is the fraction of real numbers .
Proposed by Nguyen Van Nho – Nghe An – Vietnam
UP.161. Find:

=
( + + )( + )

Proposed by Ekpo Samuel – Nigeria


UP.162. If tetrahedron; = ; = ; = ; = ; = ; =
then:

97 ROMANIAN MATHEMATICAL MAGAZINE NR. 22


Romanian Mathematical Society-Mehedinți Branch 2019

+ ≥ √ [ ]

[ ] – total area of tetrahedron .


Proposed by Daniel Sitaru – Romania
UP.163. Let , , be positive real numbers such that:
( + )( + )( + ) = . Find the minimum value of:

= + +
( + + )( + ) ( + + )( + ) ( + + )( + )
Proposed by Hoang Le Nhat Tung – Hanoi – Vietnam

UP.164. Solve the equation: (


+ )+ √ − = −
Proposed by Hoang Le Nhat Tung – Hanoi – Vietnam
UP.165. Solve the system of equations:
⎧ + + =
√ √ √
⎨ ( + + )
⎩ + + =
Proposed by Hoang Le Nhat Tung – Hanoi – Vietnam
All solutions for proposed problems can be finded on the
http//:www.ssmrmh.ro which is the adress of Romanian Mathematical
Magazine-Interactive Journal.

98 ROMANIAN MATHEMATICAL MAGAZINE NR. 22


Romanian Mathematical Society-Mehedinți Branch 2019

INDEX OF AUTHORS RMM-22


Nr.crt. Numele și prenumele Nr.crt. Numele și prenumele
1 DANIEL SITARU-ROMANIA 28 SRINIVASA RAGHAVA-INDIA
2 D.M.BĂTINEȚU-GIURGIU-ROMANIA 29 NGUYEN VAN NHO-VIETNAM
3 CLAUDIA NĂNUȚI-ROMANIA 30 HUNG NGUYEN VIET-VIETNAM
4 NECULAI STANCIU-ROMANIA 31 PHAM QUOC SANG-VIETNAM
5 MARIAN URSĂRESCU-ROMANIA 32 ARAFAT RAHMAN AKIB-BANGLADESH
6 BOGDAN FUSTEI-ROMANIA 33 NGUYEN NGOC TU-VIETNAM
7 DAN NĂNUȚI-ROMANIA 34 GEORGE APOSTOLOPOULOS-GREECE
8 MARIN CHIRCIU-ROMANIA 35 HOANG LE NHAT TUNG-VIETNAM
9 TITU ZVONARU-ROMANIA 36 MEHMET SAHIN-TURKEY
10 TUTESCU LUCIAN-ROMANIA 37 ROVSEN PIRGULIEV-AZERBAIJAN
11 ANDREI ȘTEFAN MIHALCEA-ROMANIA 38 SHIVAM SHARMA-INDIA
12 VASILE MIRCEA POPA-ROMANIA 39 ADIL ABDULLAYEV-AZERBAIJAN
13 CREMENEANU LUIZA-ROMANIA 40 DO HUU DUC THINH-VIETNAM
14 MIHALY BENCZE-ROMANIA 41 NGUYEN PHUC TANG-VIETNAM
15 PRUNARU CONSTANTINA-ROMANIA 42 MARTIN LUKAREVSKI-MACEDONIA
16 DAN NEDEIANU-ROMANIA 43 MICHEL BATAILLE-FRANCE
17 GHEORGHE ALEXE-ROMANIA 44 NGUYEN NGOC TU-VIETNAM
18 SERBAN GEORGE FLORIN-ROMANIA 45 SEYRAN IBRAHIMOV-AZERBAIJAN
19 BUTARU ZIZI-ROMANIA 46 LE KHANH SY-VIETNAM
20 BETIU ANICUTA-ROMANIA 47 NAREN BHANDARI-NEPAL
21 IOAN CORNEL VALEAN-ROMANIA 48 URFAN ALIYEV-AZERBAIJAN
22 ZAHARIA BURGHELEA-ROMANIA 49 FETI SINANI-KOSOVO
23 IONICA CONSTANTIN-ROMANIA 50 ABDUL MUKHTAR-NIGERIA
24 CARMEN CHIRFOT-ROMANIA 51 ABDALLAH EL FARISSI-ALGERIE
25 DO QUOC CHINH-VIETNAM 52 EKPO SAMUEL-NIGERIA
26 KHALEF RUHEMI-JORDAN 53 SOUMITRA MANDAL-INDIA
27 AHMED ALBAW-JORDAN

NOTĂ: Pentru a publica probleme propuse, articole și note matematice în


RMM puteți trimite materialele pe mailul: dansitaru63@yahoo.com
All solutions for proposed problems can be finded on the
http//:www.ssmrmh.ro which is the adress of Romanian Mathematical
Magazine-Interactive Journal.

99 ROMANIAN MATHEMATICAL MAGAZINE NR. 22

Potrebbero piacerti anche